Test 4 NURS 327

¡Supera tus tareas y exámenes ahora con Quizwiz!

A nurse is educating a client about the benefits of fruit versus fruit juice in the diabetic diet. The client states, "What difference does it make if you drink the juice or eat the fruit? It is all the same." What are the best responses by the nurse? Select all that apply. "Eating the fruit instead of drinking juice decreases the glycemic index by slowing absorption." "Eating the fruit is more satisfying than drinking the juice. You will get full faster." "Eating the fruit will give you more vitamins and minerals than the juice will." "Eating the fruit will lead to hyperglycemia and the fruit juice will not lead to hyperglycemia." "The fruit has less sugar than the juice."

"Eating the fruit instead of drinking juice decreases the glycemic index by slowing absorption." "The fruit has less sugar than the juice." Explanation: Eating whole fruit instead of drinking juice decreases the glycemic index, because fiber in the fruit slows absorption. A serving of juice has more sugar than a serving of fruit. Whether a fruit is more satisfying and has more vitamins and minerals than the fruit's juice are dependent on the types of fruit and juices. Eating fruit does not lead to hyperglycemia.

Which statement best indicates that a client understands how to administer his own insulin injections? "I need to be sure no air bubbles remain." "I need to wash my hands before I give myself my injection." "If I'm not feeling well, I can get a friend or neighbor to help me." "I wrote down the steps in case I forget what to do."

"I wrote down the steps in case I forget what to do." Explanation: The fact that the client has written down each step of insulin administration provides the best assurance that he'll follow through with all the proper steps. Awareness of air bubbles and hand washing indicate that the client understands certain aspects of giving an injection, but doesn't confirm he understands all of the steps. Saying that he can ask a friend or neighbor for help indicates a need for further instruction.

A physician orders blood glucose levels every 4 hours for a 4-year-old child with brittle type 1 diabetes. The parents are worried that drawing so much blood will traumatize their child. How can the nurse best reassure the parents? "Your child will need less blood work as his glucose levels stabilize." "Your child is young and will soon forget this experience." "I'll see if the physician can reduce the number of blood draws." "Our laboratory technicians use tiny needles and they're really good with children."

"Your child will need less blood work as his glucose levels stabilize." Explanation: Telling the parents that the number of blood draws will decrease as their child's glucose levels stabilize engages them in the learning process and gives them hope that the present discomfort will end as the child's condition improves. Telling the parents that their child won't remember the experience disregards their concerns and anxiety. The nurse shouldn't offer to ask the physician if he can reduce the number of blood draws; the physician needs the laboratory results to monitor the child's condition properly. Although telling the parents that the laboratory technicians are gentle and use tiny needles may be reassuring, it isn't the most appropriate response.

A female patient with diabetes who weighs 150 pounds has an ideal body weight of 118 pounds. She can lose 1 pound per week and drop her extra 32 pounds in approximately 8 months. To meet this goal, the nurse advises the patient to decrease her calories by: 2,000 per week. 2,500 per week. 3,000 per week. 3,500 per week.

3,500 per week. Explanation: A person needs to decrease caloric intake by 3,500 for each lb of weight that is lost. To lose 1 lb per week, a person would decrease his or her daily caloric intake by 500 calories (500 calories × 7 days = 3,500 calories = 1 lb).

A patient is diagnosed with overactivity of the adrenal medulla. What epinephrine value does the nurse recognize is a positive diagnostic indicator for overactivity of the adrenal medulla? 50 pg/mL 100 pg/mL 100 to 300 pg/mL 450 pg/mL

450 pg/mL Explanation: Normal plasma values of epinephrine are 100 pg/mL (590 pmol/L); normal values of norepinephrine are generally less than 100 to 550 pg/mL (590 to 3,240 pmol/L). Values of epinephrine greater than 400 pg/mL (2,180 pmol/L) or norepinephrine values greater than 2,000 pg/mL (11,800 pmol/L) are considered diagnostic of pheochromocytoma (associated with overactivity of the adrenal medulla). Values that fall between normal levels and those diagnostic of pheochromocytoma indicate the need for further testing.

Which of the following medications is considered a glitazone? Actos Glucophage Glucovance Glucophase

Actos Explanation: Pioglitazone (Actos) and rosiglitazone (Avandia) are classified as a glitazone or thiazolidinedione. Metformin (Glucophage, Glucophase) and metformin with glyburide (Glucovance) are classified as biguanides.

What is the most common cause of hyperaldosteronism? Excessive sodium intake A pituitary adenoma Deficient potassium intake An adrenal adenoma

An adrenal adenoma Explanation: An autonomous aldosterone-producing adenoma is the most common cause of hyperaldosteronism. Hyperplasia is the second most frequent cause. Aldosterone secretion is independent of sodium and potassium intake and pituitary stimulation.

During physical examination of a client with a suspected endocrine disorder, the nurse assesses the body structures. The nurse gathers this data based on the understanding that it is an important aid in which of the following? Detecting evidence of hormone hypersecretion Detecting information about possible tumor growth Determining the presence or absence of testosterone levels Determining the size of the organs and location

Detecting evidence of hormone hypersecretion Explanation: The evaluation of body structures helps the nurse detect evidence of hypersecretion or hyposecretion of hormones. This helps in the assessment of findings that are unique to specific endocrine glands. Radiographs of the chest or abdomen are taken to detect tumors. Radiographs also determine the size of the organ and its location.

An obese Hispanic client, age 65, is diagnosed with type 2 diabetes. Which statement about diabetes mellitus is true? Nearly two-thirds of clients with diabetes mellitus are older than age 60. Diabetes mellitus is more common in Hispanics and Blacks than in Whites. Type 2 diabetes mellitus is less common than type 1 diabetes mellitus. Approximately one-half of the clients diagnosed with type 2 diabetes are obese.

Diabetes mellitus is more common in Hispanics and Blacks than in Whites. Explanation: Diabetes mellitus is more common in Hispanics and Blacks than in Whites. Only about one-third of clients with diabetes mellitus are older than age 60 and 85% to 90% have type 2. At least 80% of clients diagnosed with type 2 diabetes mellitus are obese.

A client has developed drug-induced hepatitis from a drug reaction to antidepressants. What treatment does the nurse anticipate the client will receive to treat the reaction? Paracentesis Liver transplantation High-dose corticosteroids Azathioprine

High-dose corticosteroids Explanation: Drug-induced hepatitis occurs when a drug reaction damages the liver. This form of hepatitis can be severe and fatal. High-dose corticosteroids usually administered first to treat the reaction. Liver transplantation may be necessary. Paracentesis would be used to withdrawal fluid for the treatment of ascites. Azathioprine (Imuran) may be used for autoimmune hepatitis.

A client has been diagnosed with nephrogenic diabetes insipidus (DI), and the physician is initiating treatment. What medication does the nurse prepare to administer for this client? Metolazone Bumetanide Furosemide Hydrochlorothiazide

Hydrochlorothiazide Explanation: The physician prescribes a thiazide diuretic, such as hydrochlorothiazide. The thiazide acts at the proximal convoluted tubule, leaving less fluid for excretion in the distal convoluted tubules, the portion affected by nephrogenic diabetes insipidus (DI). Consequently, the client excretes water, but the total volume is less than in an untreated state. The other diuretics listed do not work on the proximal convoluted tubule and would not be effective in treatment.

Exercise lowers blood glucose levels. Which of the following are the physiologic reasons that explain this statement. Select all that apply. Increases lean muscle mass Increases resting metabolic rate as muscle size increases Decreases the levels of high-density lipoproteins Decreases total cholesterol Increases glucose uptake by body muscles

Increases lean muscle mass Increases resting metabolic rate as muscle size increases Decreases total cholesterol Increases glucose uptake by body muscles Explanation: All of the options are benefits of exercise except the effect of decreasing the levels of HDL. Exercise increases the levels of HDL.

A group of students are reviewing the various types of drugs that are used to treat diabetes mellitus. The students demonstrate understanding of the material when they identify which of the following as an example of an alpha-glucosidase inhibitor? Metformin Glyburide Miglitol Rosiglitazone

Miglitol Explanation: Alpha-glucosidase inhibitors include drugs such as miglitol and acarbose. Metformin is a biguanide. Glyburide is a sulfonylurea. Rosiglitazone is a thiazolidinedione.

The nurse is caring for a client who has an excess amount of potassium being excreted and has a serum level of 6.2 mEq/L. What group of adrenal hormones is likely to be impacting the laboratory result? Mineralocorticoids Glucocorticoids Testosterone Estrogen

Mineralocorticoids Explanation: Mineralocorticoids, primarily aldosterone, maintain water and electrolyte balances. The androgenic hormones convert to testosterone and estrogens. Glucocorticoids, such as cortisol, affect body metabolism, suppress inflammation, and help the body withstand stress.

A client has undergone a liver biopsy. Which postprocedure position is appropriate? On the left side Trendelenburg On the right side High Fowler

On the right side Explanation: In this position, the liver capsule at the site of penetration is compressed against the chest wall, and the escape of blood or bile through the perforation made for the biopsy is impeded. Positioning the client on his left side is not indicated. Positioning the client in the Trendelenburg position may be indicated if the client is in shock, but is not the position designed for the client after liver biopsy. The high Fowler position is not indicated for the client after liver biopsy.

The nurse is caring for a patient with acute pancreatitis. The patient has an order for an anticholinergic medication. The nurse explains that the patient will be receiving that medication for what reason? To decrease metabolism To depress the central nervous system and increase the pain threshold To reduce gastric and pancreatic secretions To relieve nausea and vomiting

To reduce gastric and pancreatic secretions Explanation: Anticholinergic medications reduce gastric and pancreatic secretion.

Which medication is used to decrease portal pressure, halting bleeding of esophageal varices? Spironolactone Vasopressin Nitroglycerin Cimetidine

Vasopressin Explanation: Vasopressin may be the initial therapy for esophageal varices because it produces constriction of the splanchnic arterial bed and decreases portal hypertension. Nitroglycerin has been used to prevent the side effects of vasopressin. Spironolactone and cimetidine do not decrease portal hypertension.

A client with adrenal insufficiency is gravely ill and presents with nausea, vomiting, diarrhea, abdominal pain, profound weakness, and headache. The client's family reports that the client has been doing strenuous yard work all day and was sweating profusely. Nursing management of this client would include observation for signs of: hyponatremia and hyperkalemia. hyponatremia and hypokalemia. hypernatremia and hyperkalemia. hypernatremia and hypokalemia.

hyponatremia and hyperkalemia. Correct response: hyponatremia and hyperkalemia. Explanation: The client's history and presenting symptoms suggest the onset of adrenal crisis. Laboratory findings that support adrenal defficiency and crisis include low serum sodium (hyponatremia) and high serum potassium (hyperkalemia) levels.

Most of the liver's metabolic functions are performed by: parenchymal cells. Kupffer cells. islets of Langerhans. canaliculi cells.

parenchymal cells. Explanation: The parenchymal cells perform most of the liver's metabolic functions.

Which symptoms will a nurse observe most commonly in clients with pancreatitis? severe, radiating abdominal pain black, tarry stools and dark urine increased and painful urination increased appetite and weight gain

severe, radiating abdominal pain Explanation: The most common symptom in clients with pancreatitis is severe midabdominal to upper abdominal pain, radiating to both sides and straight to the back.

A 1200-calorie diet and exercise are prescribed for a client with newly diagnosed type 2 diabetes. The nurse is teaching the client about meal planning using exchange lists. The teaching is determined to be effective based on which statement by the client? "For dinner I ate a 3-ounce hamburger on a bun, with ketchup, pickle, and onion; a green salad with 1 teaspoon Italian dressing; 1 cup of watermelon; and a diet soda." "For dinner I ate 2 cups of cooked pasta with 3-ounces of boiled shrimp, 1 cup plum tomatoes, half a cup of peas in a garlic-wine sauce, 2 cups fresh strawberries, and ice water with lemon." "For dinner I ate 4-ounces of sliced roast beef on a bagel with lettuce, tomato, and onion; 1 ounce low-fat cheese; 1 tablespoon mayonnaise; 1 cup fresh strawberry shortcake; and unsweetened iced tea." "For dinner I ate 2 ounces of sliced turkey, 1 cup mashed sweet potatoes, half a cup of carrots, half a cup of peas, a 3-ounce dinner roll, 1 medium banana, and a diet soda."

"For dinner I ate a 3-ounce hamburger on a bun, with ketchup, pickle, and onion; a green salad with 1 teaspoon Italian dressing; 1 cup of watermelon; and a diet soda." Explanation: There are six main exchange lists: bread/starch, vegetable, milk, meat, fruit, and fat. Foods within one group (in the portion amounts specified) contain equal numbers of calories and are approximately equal in grams of protein, fat, and carbohydrate. Meal plans can be based on a recommended number of choices from each exchange list. Foods on one list may be interchanged with one another, allowing for variety while maintaining as much consistency as possible in the nutrient content of foods eaten. For example, 2 starch = 2 slices bread or a hamburger bun, 3 meat = 3 oz lean beef patty, 1 vegetable = green salad, 1 fat = 1 tbsp salad dressing, 1 fruit = 1 cup watermelon; "free" items like diet soda are optional.

A client diagnosed with hyperosmolar hyperglycemic nonketotic syndrome (HHNS) is stabilized and prepared for discharge. When preparing the client for discharge and home management, which statement indicates that the client understands his condition and how to control it? "I can avoid getting sick by not becoming dehydrated and by paying attention to my need to urinate, drink, or eat more than usual." "If I experience trembling, weakness, and headache, I should drink a glass of soda that contains sugar." "I will have to monitor my blood glucose level closely and notify the physician if it's constantly elevated." "If I begin to feel especially hungry and thirsty, I'll eat a snack high in carbohydrates."

"I can avoid getting sick by not becoming dehydrated and by paying attention to my need to urinate, drink, or eat more than usual." Explanation: The client stating that he'll remain hydrated and pay attention to his eating, drinking, and voiding needs indicates understanding of HHNS. Inadequate fluid intake during hyperglycemic episodes commonly leads to HHNS. By recognizing the signs of hyperglycemia (polyuria, polydipsia, and polyphagia) and increasing fluid intake, the client may prevent HHNS. Drinking a glass of nondiet soda would be appropriate for hypoglycemia. A client whose diabetes is controlled with oral antidiabetic agents usually doesn't need to monitor blood glucose levels. A high-carbohydrate diet would exacerbate the client's condition, particularly if fluid intake is low.

A nurse prepares teaching for a client with newly-diagnosed diabetes. Which statements about the role of insulin will the nurse include in the teaching? Select all that apply. "Insulin permits entry of glucose into the cells of the body." "Insulin promotes synthesis of proteins in various body tissues." "Insulin promotes the storage of fat in adipose tissue." "Insulin interferes with glucagon from the pancreas." "Insulin interferes with the release of growth hormone from the pituitary."

"Insulin permits entry of glucose into the cells of the body." "Insulin promotes synthesis of proteins in various body tissues." "Insulin promotes the storage of fat in adipose tissue." Explanation: Insulin is a hormone secreted by the endocrine part of the pancreas. In addition to lowering blood glucose by permitting entry of glucose into the cells, insulin also promotes protein synthesis and the storage of fat in adipose tissue. Somatostatin exerts a hypoglycemic effect by interfering with glucagon from the pancreas and the release of growth hormone from the pituitary.

A client discharged after a laparoscopic cholecystectomy calls the surgeon's office reporting severe right shoulder pain 24 hours after surgery. Which statement is the correct information for the nurse to provide to this client? "This pain is caused from the gas used to inflate your abdominal area during surgery. Sitting upright in a chair, walking, or using a heating pad may ease the discomfort." "This pain is caused from your incision. Take analgesics as needed and as prescribed and report to the surgeon if pain is unrelieved even with analgesic use." "This may be the initial symptoms of an infection. You need to come to see the surgeon today for an evaluation." "This pain may be caused by a bile duct injury. You will need to go to the hospital immediately to have this evaluated."

"This pain is caused from the gas used to inflate your abdominal area during surgery. Sitting upright in a chair, walking, or using a heating pad may ease the discomfort." Explanation: If pain occurs in the right shoulder or scapular area (from migration of the carbon dioxide used to insufflate the abdominal cavity during the procedure), the nurse may recommend using a heating pad for 15 to 20 minutes hourly, sitting up in a bed or chair, or walking.

A client is evaluated for severe pain in the right upper abdominal quadrant, which is accompanied by nausea and vomiting. The physician diagnoses acute cholecystitis and cholelithiasis. For this client, which nursing diagnosis takes top priority? Acute pain related to biliary spasms Deficient knowledge related to prevention of disease recurrence Anxiety related to unknown outcome of hospitalization Imbalanced nutrition: Less than body requirements related to biliary inflammation

Acute pain related to biliary spasms Explanation: The chief symptom of cholecystitis is abdominal pain or biliary colic. Typically, the pain is so severe that the client is restless and changes positions frequently to find relief. Therefore, the nursing diagnosis of Acute pain related to biliary spasms takes highest priority. Until the acute pain is relieved, the client can't learn about prevention, may continue to experience anxiety, and can't address nutritional concerns.

Which statement about fluid replacement is accurate for a client with hyperosmolar hyperglycemic nonketotic syndrome? Administer 2 to 3 L of IV fluid rapidly. Administer 10 L of IV fluid over the first 24 hours. Administer a dextrose solution containing normal saline solution. Administer IV fluid slowly to prevent circulatory overload and collapse.

Administer 2 to 3 L of IV fluid rapidly. Explanation: Regardless of the client's medical history, rapid fluid resuscitation is critical for maintaining cardiovascular integrity. Profound intravascular depletion requires aggressive fluid replacement. A typical fluid resuscitation protocol is 6 L of fluid over the first 12 hours, with more fluid to follow over the next 24 hours. Various fluids can be used, depending on the degree of hypovolemia. Commonly ordered fluids include dextran (in cases of hypovolemic shock), isotonic normal saline solution and, when the client is stabilized, hypotonic half-normal saline solution.

A client with type 1 diabetes must undergo bowel resection in the morning. How should the nurse proceed while caring for him on the morning of surgery? Administer half of the client's typical morning insulin dose as ordered. Administer an oral antidiabetic agent as ordered. Administer an I.V. insulin infusion as ordered. Administer the client's normal daily dose of insulin as ordered.

Administer half of the client's typical morning insulin dose as ordered. Explanation: If the nurse administers the client's normal daily dose of insulin while he's on nothing-by-mouth status before surgery, he'll experience hypoglycemia. Therefore, the nurse should administer half the daily insulin dose as ordered. Oral antidiabetic agents aren't effective for type 1 diabetes. I.V. insulin infusions aren't necessary to manage blood glucose levels in clients undergoing routine surgery.

A client newly diagnosed with type 1 diabetes has an unusual increase in blood glucose from bedtime to morning. The physician suspects the client is experiencing insulin waning. Based on this diagnosis, the nurse expects which change to the client's medication regimen? Administering a dose of intermediate-acting insulin before the evening meal Changing the time of evening injection of intermediate-acting insulin from dinnertime to bedtime Decreasing evening bedtime dose of intermediate-acting insulin and administering a bedtime snack Increasing morning dose of long-acting insulin

Administering a dose of intermediate-acting insulin before the evening meal Explanation: Insulin waning is a progressive rise in blood glucose form bedtime to morning. Treatment includes increasing the evening (before dinner or bedtime) dose of intermediate-acting or long-acting insulin or instituting a dose of insulin before the evening meal if that is not already part of the treatment regimen.

Which term is used to describe a chronic liver disease in which scar tissue surrounds the portal areas? Alcoholic cirrhosis Postnecrotic cirrhosis Biliary cirrhosis Compensated cirrhosis

Alcoholic cirrhosis Explanation: This type of cirrhosis is due to chronic alcoholism and is the most common type of cirrhosis. In postnecrotic cirrhosis, there are broad bands of scar tissue, which are a late result of a previous acute viral hepatitis. In biliary cirrhosis, scarring occurs in the liver around the bile ducts. Compensated cirrhosis is a general term given to the state of liver disease in which the liver continues to be able to function effectively.

A nurse has been caring for a client newly diagnosed with diabetes mellitus. The client is overwhelmed by what he's facing and not sure he can handle giving himself insulin. This client has been discharged and the charge nurse is insisting the nurse hurry because she needs the space for clients being admitted. How should the nurse handle the situation? Suggest the client find a supportive friend or family member to assist in his care. Ask the physician to delay the discharge because the client requires further teaching. Tell the charge nurse she doesn't believe this client will be safe and refuse to rush. Ask the physician for a referral for a diabetes nurse-educator to see the client before discharge.

Ask the physician to delay the discharge because the client requires further teaching. Explanation: The nurse's primary concern should be the safety of the client after discharge. She should provide succinct information to the physician concerning the client's needs, express her concern about ensuring the client's safety, and ask the physician to delay the client's discharge. The nurse shouldn't suggest that the client rely on a friend or family member because she doesn't know if a friend or family member will be available to help. Refusing to rush and telling the charge nurse she isn't sure the client will be safe demonstrate appropriate intentions, but these actions don't alleviate the pressure to discharge the client. Asking a physician to refer the client to a diabetic nurse-educator addresses the client's needs, but isn't the best response because there's no guarantee a diabetic nurse-educator will be available on such short notice.

Which symptom of thyroid disease is seen in older adults? Restlessness Hyperactivity Atrial fibrillation Weight gain

Atrial fibrillation Explanation: Symptoms seen in older adults include weight loss and atrial fibrillation. Older adults may not experience restlessness or hyperactivity.

A client with diabetic ketoacidosis was admitted to the intensive care unit 4 hours ago and has these laboratory results: blood glucose level 450 mg/dl, serum potassium level 2.5 mEq/L, serum sodium level 140 mEq/L, and urine specific gravity 1.025. The client has two IV lines in place with normal saline solution infusing through both. Over the past 4 hours, his total urine output has been 50 ml. Which physician order should the nurse question? Infuse 500 ml of normal saline solution over 1 hour. Hold insulin infusion for 30 minutes. Add 40 mEq potassium chloride to an infusion of half normal saline solution and infuse at a rate of 10 mEq/hour. Change the second IV solution to dextrose 5% in water.

Change the second IV solution to dextrose 5% in water. Explanation: The nurse should question the physician's order to change the second IV solution to dextrose 5% in water. The client should receive normal saline solution through the second IV site until his blood glucose level reaches 250 mg/dl. The client should receive a fluid bolus of 500 ml of normal saline solution. The client's urine output is low and his specific gravity is high, which reveals dehydration. The nurse should expect to hold the insulin infusion for 30 minutes until the potassium replacement has been initiated. Insulin administration causes potassium to enter the cells, which further lowers the serum potassium level. Further lowering the serum potassium level places the client at risk for life-threatening cardiac arrhythmias.

Increased appetite and thirst may indicate that a client with chronic pancreatitis has developed diabetes melitus. Which of the following explains the cause of this secondary diabetes? Dysfunction of the pancreatic islet cells Ingestion of foods high in sugar Inability for the liver to reabsorb serum glucose Renal failure

Dysfunction of the pancreatic islet cells Explanation: Diabetes mellitus resulting from dysfunction of the pancreatic islet cells is treated with diet, insulin, or oral antidiabetic agents. The hazard of severe hypoglycemia with alcohol consumption is stressed to the client and family. When secondary diabetes develops in a client with chronic pancreatitis, the client experiences increased appetite, thirst, and urination. A standard treatment with pancreatitis is to make the client NPO. The dysfunction is related to the pancreas, not the liver.

A nurse is preparing to palpate a client's thyroid gland. Which action by the nurse is appropriate? Have the client flex his neck onto his chest and cough while she palpates the anterior neck with her fingertips. Place her hands around the client's neck, with the thumbs in the front of the neck, and gently massage the anterior neck. Encircle the client's neck with both hands, have the client slightly extend his neck, and ask him to swallow. Have the client hyperextend his neck and take slow, deep inhalations while she palpates his neck with her fingertips.

Encircle the client's neck with both hands, have the client slightly extend his neck, and ask him to swallow. Explanation: When palpating the thyroid gland, the nurse should encircle the client's neck with both hands, have the client slightly extend his neck, and ask him to swallow. As the client swallows, the gland is palpated for enlargement as the tissue rises and falls. Having the client flex his neck wouldn't allow for palpation. Massaging the area or checking during inhalation doesn't allow for the movement of tissue that swallowing provides.

A group of students is reviewing information about the liver and associated disorders. The group demonstrates understanding of the information when they identify which of the following as a primary function of the liver? Breakdown amino acids Convert urea into ammonia Excrete bile Break down coagulation factors

Excrete bile Explanation: The liver forms and excretes bile, synthesizes amino acids from the breakdown of proteins, converts ammonia into urea, and synthesizes the factors needed for blood coagulation.

The nurse is aware that the clinical symptoms of a patient with hypoparathyroidism are the result of the initial physiologic response of: Hypocalcemia. Decreased levels of vitamin D. Increased serum levels of phosphate. Cardiac arrhythmias.

Hypocalcemia. Explanation: Hypoparathyroidism results in hypocalcemia, which triggers a series of physiologic responses, including the choices presented.

Which combination of adverse effects should a nurse monitor for when administering IV insulin to a client with diabetic ketoacidosis? Hypokalemia and hypoglycemia Hypocalcemia and hyperkalemia Hyperkalemia and hyperglycemia Hypernatremia and hypercalcemia

Hypokalemia and hypoglycemia Explanation: Blood glucose needs to be monitored in clients receiving IV insulin because of the risk of hyperglycemia or hypoglycemia. Hypoglycemia might occur if too much insulin is administered. Hypokalemia, not hyperkalemia, might occur because I.V. insulin forces potassium into cells, thereby lowering the plasma level of potassium. Calcium and sodium levels aren't affected by IV insulin administration.

When reviewing laboratory results for a patient with a possible diagnosis of hypoparathyroidism, the nurse knows that this condition is characterized by which of the following? Inadequate secretion of parathormone Increase in serum calcium Lowered blood phosphate Increase in the renal excretion of phosphate

Inadequate secretion of parathormone Explanation: In hypoparathyroidism, there is an increased blood phosphate. Blood calcium is decreased, and there is a decreased renal excretion of phosphate. The secretion of parathormone is inadequate.

A nurse is assessing a client who is receiving total parenteral nutrition (TPN). Which finding suggests that the client has developed hyperglycemia? Cheyne-Stokes respirations Increased urine output Decreased appetite Diaphoresis

Increased urine output Explanation: Glucose supplies most of the calories in TPN; if the glucose infusion rate exceeds the client's rate of glucose metabolism, hyperglycemia arises. When the renal threshold for glucose reabsorption is exceeded, osmotic diuresis occurs, causing an increased urine output. A decreased appetite and diaphoresis suggest hypoglycemia, not hyperglycemia. Cheyne-Stokes respirations are characterized by a period of apnea lasting 10 to 60 seconds, followed by gradually increasing depth and frequency of respirations. Cheyne-Stokes respirations typically occur with cerebral depression or heart failure.

A patient is diagnosed with a deficiency in vasopressin, a posterior pituitary hormone. Therefore, a primary nursing responsibility is to assess for: Indicators of dehydration. Glycosuria Serum calcium levels. Indicators of hyponatremia.

Indicators of dehydration. Explanation: A deficiency in vasopressin, also known as the antidiuretic hormone, would result in increased urinary output, thirst, and dehydration. No glucose is lost in the urine. Hypernatremia occurs with dehydration.

Which factor presents the most likely cause for weight gain in a diabetic client who is controlled with insulin? Insulin is an anabolic hormone. Insulin provides more efficient use of glucose. Faulty fat metabolism is shut off. Weight gain is attributed to fluid retention.

Insulin is an anabolic hormone. Explanation: Insulin is an anabolic hormone that is known to cause weight gain. Insulin does lower blood glucose levels by allowing for active transport of glucose into the cells. Faulty fat and protein metabolism will cease once glucose provides the needed the fuel for energy. The restoration of normal metabolism is not the primary cause for weight gain in a client prescribed insulin. Fluid retention is not indicated in this client.

Which term refers to the progressive increase in blood glucose from bedtime to morning? Somogyi effect Insulin waning Dawn phenomenon Diabetic ketoacidosis (DKA)

Insulin waning Explanation: Insulin waning is a progressive rise in blood glucose from bedtime to morning. The dawn phenomenon occurs when blood glucose is relatively normal until about 3 a.m., when the level begins to rise. The Somogyi effect occurs when blood glucose is normal or elevated at bedtime, decreases at 2 to 3 a.m. to hypoglycemia levels, and subsequently increases as a result of the production of counter-regulatory hormones. DKA is caused by an absence or markedly inadequate amount of insulin. This insulin deficit results in disorders in the metabolism of carbohydrates, proteins, and fats. The primary clinical features of DKA are hyperglycemia, ketosis, dehydration, electrolyte loss, and acidosis.

While conducting a physical examination of a client, which of the following skin findings would alert the nurse to the possibility of liver problems? Select all that apply. Jaundice Petechiae Ecchymoses Cyanosis of the lips Aphthous stomatitis

Jaundice Petechiae Ecchymoses Explanation: The skin, mucosa, and sclerae are inspected for jaundice. The nurse observes the skin for petechiae or ecchymotic areas (bruises), spider angiomas, and palmar erythema. Cyanosis of the lips is indicative of a problem with respiratory or cardiovascular dysfunction. Aphthous stomatitis is a term for mouth ulcers and is a gastrointestinal abnormal finding.

A patient is diagnosed with type 1 diabetes. What clinical characteristics does the nurse expect to see in this patient? Select all that apply. Ketosis-prone Little endogenous insulin Obesity at diagnoses Younger than 30 years of age Older than 65 years of age

Ketosis-prone Little endogenous insulin Younger than 30 years of age Explanation: Type I diabetes mellitus is associated with the following characteristics: onset any age, but usually young (<30 y); usually thin at diagnosis, recent weight loss; etiology includes genetic, immunologic, and environmental factors (e.g., virus); often have islet cell antibodies; often have antibodies to insulin even before insulin treatment; little or no endogenous insulin; need exogenous insulin to preserve life; and ketosis prone when insulin absent.

The nurse is assessing a patient with nonproliferative (background) retinopathy. When examining the retina, what would the nurse expect to assess? Select all that apply. Leakage of fluid or serum (exudates) Microaneurysms Focal capillary single closure Detachment Blurred optic discs

Leakage of fluid or serum (exudates) Microaneurysms Focal capillary single closure Explanation: Almost all patients with type 1 diabetes and the majority of patients with type 2 diabetes have some degree of retinopathy after 20 years (ADA, 2013). Changes in the microvasculature include microaneurysms, intraretinal hemorrhage, hard exudates, and focal capillary closure.

A nurse obtains a fingerstick glucose level of 45 mg/dl on a client newly diagnosed with diabetes mellitus. The client is alert and oriented, and the client's skin is warm and dry. How should the nurse intervene? Give the client 4 oz of milk and a graham cracker with peanut butter. Obtain a serum glucose level. Obtain a repeat fingerstick glucose level. Notify the physician.

Obtain a repeat fingerstick glucose level. Explanation: The nurse should recheck the fingerstick glucose level to verify the original result because the client isn't exhibiting signs of hypoglycemia. The nurse should give the client milk and a graham cracker with peanut butter or a glass of orange juice after confirming the low glucose level. It isn't necessary to notify the physician or to obtain a serum glucose level at this time.

A client is actively bleeding from esophageal varices. Which medication would the nurse mostexpect to be administered to this client? Octreotide Spironolactone Propranolol Lactulose

Octreotide Explanation: In an actively bleeding client, medications are administered initially because they can be obtained and administered quicker than other therapies. Octreotide (Sandostatin) causes selective splanchnic vasoconstriction by inhibiting glucagon release and is used mainly in the management of active hemorrhage. Propranolol (Inderal) and nadolol (Corgard), beta-blocking agents that decrease portal pressure, are the most common medications used both to prevent a first bleeding episode in clients with known varices and to prevent rebleeding. Beta-blockers should not be used in acute variceal hemorrhage, but they are effective prophylaxis against such an episode. Spironolactone (Aldactone), an aldosterone-blocking agent, is most often the first-line therapy in clients with ascites from cirrhosis. Lactulose (Cephulac) is administered to reduce serum ammonia levels in clients with hepatic encephalopathy.

A client has undergone a liver biopsy. After the procedure, the nurse should place the client in which position? On the left side Trendelenburg On the right side High Fowler

On the right side Explanation: Immediately after the biopsy, assist the client to turn on to the right side; place a pillow under the costal margin, and caution the client to remain in this position. In this position, the liver capsule at the site of penetration is compressed against the chest wall, and the escape of blood or bile through the perforation made for the biopsy is impeded. Positioning the client on the left side is not indicated. Positioning the client in the Trendelenburg position may be indicated if the client is in shock, but it is not the position designed for the client after liver biopsy. The high Fowler position is not indicated for the client after liver biopsy.

A client diagnosed with acute pancreatitis is being transferred to another facility. The nurse caring for the client completes the transfer summary, which includes information about the client's drinking history and other assessment findings. Which assessment findings confirm the diagnosis of acute pancreatitis? Pain with abdominal distention and hypotension Presence of blood in the client's stool and recent hypertension Presence of easy bruising and bradycardia Adventitious breath sounds and hypertension

Pain with abdominal distention and hypotension Explanation: Assessment findings associated with pancreatitis include pain with abdominal distention and hypotension. Blood in stools and recent hypertension aren't associated with pancreatitis; fatty diarrhea and hypotension are usually present. Presence of easy bruising and bradycardia aren't found with pancreatitis; the client typically experiences tachycardia, not bradycardia. Adventitious breath sounds and hypertension aren't associated with pancreatitis.

A client who was diagnosed with type 1 diabetes 14 years ago is admitted to the medical-surgical unit with abdominal pain. On admission, the client's blood glucose level is 470 mg/dl. Which finding is most likely to accompany this blood glucose level? Cool, moist skin Rapid, thready pulse Arm and leg trembling Slow, shallow respirations

Rapid, thready pulse Explanation: This client's abnormally high blood glucose level indicates hyperglycemia, which typically causes polyuria, polyphagia, and polydipsia. Because polyuria leads to fluid loss, the nurse should expect to assess signs of deficient fluid volume, such as a rapid, thready pulse; decreased blood pressure; and rapid respirations. Cool, moist skin and arm and leg trembling are associated with hypoglycemia. Rapid respirations — not slow, shallow ones — are associated with hyperglycemia.

A client with pancreatitis is admitted to the medical intensive care unit. Which nursing intervention is most appropriate? Providing generous servings at mealtime Reserving an antecubital site for a peripherally inserted central catheter (PICC) Providing the client with plenty of P.O. fluids Limiting I.V. fluid intake according to the physician's order

Reserving an antecubital site for a peripherally inserted central catheter (PICC) Explanation: Pancreatitis treatment typically involves resting the GI tract by maintaining nothing-by-mouth status. The nurse should reserve the antecubital site for a PICC, which enables the client to receive long-term total parenteral nutrition. Clients in the acute stages of pancreatitis also require large volumes of I.V. fluids to compensate for fluid loss.

A client is transferred to a rehabilitation center after being treated in the hospital for a stroke. Because the client has a history of Cushing's syndrome (hypercortisolism) and chronic obstructive pulmonary disease, the nurse formulates a nursing diagnosis of: Risk for imbalanced fluid volume related to excessive sodium loss. Risk for impaired skin integrity related to tissue catabolism secondary to cortisol hypersecretion. Ineffective health maintenance related to frequent hypoglycemic episodes secondary to Cushing's syndrome. Decreased cardiac output related to hypotension secondary to Cushing's syndrome.

Risk for impaired skin integrity related to tissue catabolism secondary to cortisol hypersecretion. Explanation: Cushing's syndrome causes tissue catabolism, resulting in thinning skin and connective tissue loss; along with immobility related to stroke, these factors increase this client's risk for impaired skin integrity. The exaggerated glucocorticoid activity in Cushing's syndrome causes sodium and water retention which, in turn, leads to edema and hypertension. Therefore, Risk for imbalanced fluid volume and Decreased cardiac output are inappropriate nursing diagnoses. Increased glucocorticoid activity also causes persistent hyperglycemia, eliminating Ineffective health maintenance related to frequent hypoglycemic episodes as an appropriate nursing diagnosis.

A client recovers from an episode of gallbladder inflammation and the client's diet is advanced from a low-fat liquid diet. Which meals will the nurse recommend to the client? Select all that apply. Roasted chicken, mashed potatoes, and green beans. Marinated lean steak, steamed rice, and roasted zucchini. Oatmeal with a cooked fruit compote and hot tea. Egg salad sandwich with low fat mayonnaise and a side salad with ranch dressing. Vegetable lasagna, bread sticks, and steamed broccoli.

Roasted chicken, mashed potatoes, and green beans. Marinated lean steak, steamed rice, and roasted zucchini. Oatmeal with a cooked fruit compote and hot tea. Explanation: Many individuals recover from an acute attack of gallbladder inflammation. After tolerating a low-fat liquid diet, the diet is usually advanced to allow cooked fruits, rice or tapioca, lean meats, mashed potatoes, and non-gas-forming vegetables. Bread, coffee, or tea may be added as tolerated. The client should avoid eggs, fatty dressings or mayonnaise, cheese, or gas-forming vegetables.

A client with symptoms of Cushing syndrome is admitted to the hospital for evaluation and treatment. The nurse is creating a plan of care for the client. Which is an appropriate nursing diagnosis? Self-care deficit related to weakness, fatigue, muscle wasting, and altered sleep patterns Insomnia related to increased nighttime voiding Impaired nutrition: more than body requirements related to polyphagia Activity intolerance related to muscle cramps, cardiac dysrhythmias, and weakness

Self-care deficit related to weakness, fatigue, muscle wasting, and altered sleep patterns Explanation: The major goals for the client include decreased risk of injury, decreased risk of infection, increased ability to perform self-care activities, improved skin integrity, improved body image, improved mental function, and absence of complications. The other nursing diagnoses do not apply in Cushing syndrome.

A nurse is teaching a client about the cause of acute pancreatitis. The nurse evaluates the teaching as effective when the client correctly identifies which condition as a cause of acute pancreatitis? Fibrosis and atrophy of the pancreatic gland Calcification of the pancreatic duct, leading to its blockage Self-digestion of the pancreas by its own proteolytic enzymes Use of loop diuretics to increase the incidence of pancreatitis

Self-digestion of the pancreas by its own proteolytic enzymes Explanation: Self-digestion of the pancreas by its own proteolytic enzymes, principally trypsin, causes acute pancreatitis. Of clients with acute pancreatitis, 80% had undiagnosed chronic pancreatitis. Gallstones enter the common bile duct and lodge at the ampulla of Vater, obstructing the flow of pancreatic juice or causing a reflux of bile from the common bile duct into the pancreatic duct, thus activating the powerful enzymes within the pancreas. Normally, these remain in an inactive form until the pancreatic secretions reach the lumen of the duodenum. Activation of the enzymes can lead to vasodilation, increased vascular permeability, necrosis, erosion, and hemorrhage.

A client is diagnosed with gallstones in the bile ducts. What laboratory results should the nurse review? Serum ammonia concentration Serum albumin concentration Serum alkaline phosphatase Serum globulin concentration

Serum alkaline phosphatase Explanation: Serum alkaline phosphatase in absence of bone disease,is used to measure biliary tract obstruction. Jaundice occurs in a few clients with gallbladder disease, usually with obstruction of the common bile duct. If the flow of bile is impeded (eg, by gallstones in the bile ducts), bilirubin does not enter the intestine. Serum ammonia levels are associated with liver disease. Serum albumin is associated with protein (nutrition) abnormalities. Serum globulin concentration are also associated with protein in the blood.

A client comes to the ED with severe abdominal pain, nausea, and vomiting. The physician plans to rule out acute pancreatitis. The nurse would expect the diagnosis to be confirmed by an elevated result on which laboratory test? Serum calcium Serum bilirubin Serum amylase Serum potassium

Serum amylase Explanation: Serum amylase and lipase concentrations are used to make the diagnosis of acute pancreatitis. Serum amylase and lipase concentrations are elevated within 24 hours of the onset of symptoms. Serum amylase usually returns to normal within 48 to 72 hours, but the serum lipase concentration may remain elevated for a longer period, often days longer than amylase. Urinary amylase concentrations also become elevated and remain elevated longer than serum amylase concentrations.

Laboratory studies indicate a client's blood glucose level is 185 mg/dl. Two hours have passed since the client ate breakfast. Which test would yield the most conclusive diagnostic information about the client's glucose use? Fasting blood glucose test 6-hour glucose tolerance test Serum glycosylated hemoglobin (Hb A1c) Urine ketones

Serum glycosylated hemoglobin (Hb A1c) Explanation: Hb A1c is the most reliable indicator of glucose use because it reflects blood glucose levels for the prior 3 months. Although a fasting blood glucose test and a 6-hour glucose tolerance test yield information about a client's use of glucose, the results are influenced by such factors as whether the client recently ate breakfast. Presence of ketones in the urine also provides information about glucose use but is limited in its diagnostic significance.

The nurse is asking the client with acute pancreatitis to describe the pain. What pain symptoms does the client describe related to acute pancreatitis? Dull pain, points to epigastric area Sharp, stabbing pain in the left lower quadrant of the abdomen Severe mid-abdominal to upper abdominal pain radiating to both sides and to the back Severe abdominal pain that radiates to the right shoulder

Severe mid-abdominal to upper abdominal pain radiating to both sides and to the back Explanation: The most common complaint of clients with pancreatitis is severe mid-abdominal to upper abdominal pain, radiating to both sides and straight to the back. The other answers are not pain that is usually associated with acute pancreatitis.

A client with acute pancreatitis reports muscle cramping in the lower extremities. What pathophysiology concept represents the reason the client is reporting this? Tetany related to hypocalcemia Muscle spasm related to hypokalemia Muscle pain related to referred pain manifestations Tetany related to hypercalcemia

Tetany related to hypocalcemia Explanation: A client with acute pancreatitis who reports muscle cramping or pain should be suspected of having hypocalcemia and tetany of the muscles. Hypocalcemia may occur in acute pancreatitis because, when auto digestion of the pancreas occurs, calcium binds to fatty acids and calcium is decreased in the blood. This is a potentially life-threatening complication of pancreatitis and needs to be immediately addressed.

A nurse is educating a client who has been treated for hepatic encephalopathy about dietary restrictions to prevent ammonia accumulation. What should the nurse include in the dietary teaching? Decrease the amount of fats in the diet. Increase the amount of potassium in the diet. The amount of protein is not restricted in the diet. Increase the amount of magnesium in the diet.

The amount of protein is not restricted in the diet. Explanation: Clients with hepatic encephalopathy and their families are advised that protein intake should not be restricted in hepatic encephalopathy, as was recommended in the past. Protein intake should be maintained at 1.2 to 1.5 g/kg per day. Electrolyte balance may need to be corrected with some clients, but there are no specific recommendations for potassium and magnesium. Fat intake is not limited in clients who have hepatic encephalopathy.

A nurse is reviewing a laboratory order for a client who is scheduled to be tested for a suspected endocrine disorder. The client was recently seen in the office for bronchitis, and you note that he is still taking cough medication. The nurse explains to the client that he will not be able to get his lab testing done today. Why has the testing been postponed? The client is being tested for a thyroid disorder The client is being tested for a parathyroid disorder The client is being tested for an adrenal disorder The client is being tested for a parathyroid disorder The client is being tested for a pituitary disorder

The client is being tested for a thyroid disorder Explanation: If a client has recently taken a drug that contains iodine (e.g., some cough medicines) or has had radiographic contrast studies that used iodine, thyroid test results may be inaccurate.

Ammonia, the major etiologic factor in the development of encephalopathy, inhibits neurotransmission. Increased levels of ammonia are damaging to the body. The largest source of ammonia is from: The digestion of dietary and blood proteins. Excessive diuresis and dehydration. Severe infections and high fevers. Excess potassium loss subsequent to prolonged use of diuretics.

The digestion of dietary and blood proteins. Explanation: Circumstances that increase serum ammonia levels tend to aggravate or precipitate hepatic encephalopathy. The largest source of ammonia is the enzymatic and bacterial digestion of dietary and blood proteins in the GI tract. Ammonia from these sources increases as a result of GI bleeding (ie, bleeding esophageal varices, chronic GI bleeding), a high-protein diet, bacterial infection, or uremia.

What life-threatening outcome should the nurse monitor for in a client who is not compliant with taking his antithyroid medication? Thyrotoxic crisis Myxedema coma Diabetes insipidus Syndrome of inappropriate antidiuretic hormone secretion

Thyrotoxic crisis Explanation: Thyrotoxic crisis, an abrupt and life-threatening form of hyperthyroidism, is thought to be triggered by extreme stress, infection, diabetic ketoacidosis, trauma, toxemia of pregnancy, or manipulation of a hyperactive thyroid gland during surgery or physical examination. Although rare, this condition may occur in clients with undiagnosed or inadequately treated hyperthyroidism. Myxedema coma is the opposite in symptoms that thyrotoxic crisis. Diabetes insipidus (DI) and syndrome of inappropriate antidiuretic hormone secretion (SIADH) clinical manifestations do not correlate with medication taken for hyperthyroidism.

The physician has written the following orders for a new client admitted with pancreatitis: bed rest, nothing by mouth (NPO), and administration of total parenteral nutrition (TPN) . Which does the nurse attribute as the reason for NPO status? To drain the pancreatic bed To aid opening up of pancreatic duct To prevent the occurrence of fibrosis To avoid inflammation of the pancreas

To avoid inflammation of the pancreas Explanation: Pancreatic secretion is increased by food and fluid intake and may cause inflammation of the pancreas.

Which factor is the focus of nutrition intervention for clients with type 2 diabetes? Protein metabolism Blood glucose level Weight loss Carbohydrate intake

Weight loss Explanation: Weight loss is the focus of nutrition intervention for clients with type 2 diabetes. A low-calorie diet may improve clinical symptoms, and even a mild to moderate weight loss, such as 10 to 20 pounds, may lower blood glucose levels and improve insulin action. Consistency in the total amount of carbohydrates consumed is considered an important factor that influences blood glucose level. Protein metabolism is not the focus of nutrition intervention for clients with type 2 diabetes.

A physician orders lactulose (Cephulac), 30 ml three times daily, when a client with cirrhosis develops an increased serum ammonia level. To evaluate the effectiveness of lactulose, the nurse should monitor: urine output. abdominal girth. stool frequency. level of consciousness (LOC).

level of consciousness (LOC). Explanation: In cirrhosis, the liver fails to convert ammonia to urea. Ammonia then builds up in the blood and is carried to the brain, causing cerebral dysfunction. When this occurs, lactulose is administered to promote ammonia excretion in the stool and thus improve cerebral function. Because LOC is an accurate indicator of cerebral function, the nurse can evaluate the effectiveness of lactulose by monitoring the client's LOC. Monitoring urine output, abdominal girth, and stool frequency helps evaluate the progress of cirrhosis, not the effectiveness of lactulose.

A client who was recently diagnosed with carcinoma of the pancreas and is having a procedure in which the head of the pancreas is removed. In addition, the surgeon will remove the duodenum and stomach, redirecting the flow of secretions from the stomach, gallbladder, and pancreas into the middle section of the small intestine. What procedure is this client having performed? radical pancreatoduodenectomy cholecystojejunostomy total pancreatectomy distal pancreatectomy

radical pancreatoduodenectomy Explanation: Radical pancreatoduodenectomy involves removing the head of the pancreas, resecting the duodenum and stomach, and redirecting the flow of secretions from the stomach, gallbladder, and pancreas into the jejunum. Cholecystojejunostomy is a rerouting of the pancreatic and biliary drainage systems, which may be done to relieve obstructive jaundice. This measure is considered palliative only. A pancreatectomy is the surgical removal of the pancreas. A pancreatectomy may be total, in which case the entire organ is removed, usually along with the spleen, gallbladder, common bile duct, and portions of the small intestine and stomach. A distal pancreatectomy is a surgical procedure to remove the bottom half of the pancreas.

Which factor is the focus of nutrition intervention for clients with type 2 diabetes? weight loss carbohydrate intake protein metabolism blood glucose level

weight loss Explanation: Weight loss is the focus of nutrition intervention for clients with type 2 diabetes. A low-calorie diet may improve clinical symptoms; even a mild to moderate weight loss such as 10 to 20 lb may lower blood glucose levels and improve insulin action.

A client with type 2 diabetes has recently been prescribed acarbose, and the nurse is explaining how to take this medication. The teaching is determined to be effective based on which statement by the client? "I will take this medication in the morning, 15 minutes before breakfast." "I will take this medication in the morning, with my first bite of breakfast." "This medication needs to be taken after the midday meal." "It does not matter what time of day I take this medication."

"I will take this medication in the morning, with my first bite of breakfast." Explanation: Alpha-glucosidase inhibitors such as acarbose and miglitol, delay absorption of complex carbohydrates in the intestine and slow entry of glucose into systemic circulation. They must be taken with the first bite of food to be effective.

A nurse is preparing a continuous insulin infusion for a child with diabetic ketoacidosis and a blood glucose level of 800 mg/dl. Which solution is the most appropriate at the beginning of therapy? 100 units of regular insulin in normal saline solution 100 units of neutral protamine Hagedorn (NPH) insulin in normal saline solution 100 units of regular insulin in dextrose 5% in water 100 units of NPH insulin in dextrose 5% in water

100 units of regular insulin in normal saline solution Explanation: Continuous insulin infusions use only short-acting regular insulin. Insulin is added to normal saline solution and administered until the client's blood glucose level falls. Further along in the therapy, a dextrose solution is administered to prevent hypoglycemia.

When caring for a client with cirrhosis, which symptoms should a nurse report immediately? Select all that apply. change in mental status signs of GI bleeding anorexia and dyspepsia diarrhea or constipation

change in mental status signs of GI bleeding Explanation: The nurse reports any change in mental status or signs of GI bleeding immediately because they indicate secondary complications.

A nurse is caring for a client with cholelithiasis. Which sign indicates obstructive jaundice? Straw-colored urine Reduced hematocrit Clay-colored stools Elevated urobilinogen in the urine

Clay-colored stools Explanation: Obstructive jaundice develops when a stone obstructs the flow of bile in the common bile duct. When the flow of bile to the duodenum is blocked, the lack of bile pigments results in a clay-colored stool. In obstructive jaundice, urine tends to be dark amber (not straw-colored) as a result of soluble bilirubin in the urine. Hematocrit levels aren't affected by obstructive jaundice. Because obstructive jaundice prevents bilirubin from reaching the intestine (where it's converted to urobilinogen), the urine contains no urobilinogen.

Which factors will cause hypoglycemia in a client with diabetes? Select all that apply. Client has not consumed food and continues to take insulin or oral antidiabetic medications. Client has not consumed sufficient calories. Client has been exercising more than usual. Client has been sleeping excessively. Client is experiencing effects of the aging process.

Client has not consumed food and continues to take insulin or oral antidiabetic medications. Client has not consumed sufficient calories. Client has been exercising more than usual. Explanation: Hypoglycemia can occur when a client with diabetes is not eating at all and continues to take insulin or oral antidiabetic medications, is not eating sufficient calories to compensate for glucose-lowering medications, or is exercising more than usual. Excessive sleep and aging are not factors in the onset of hypoglycemia.

Which diagnostic test is done to determine a suspected pituitary tumor? Radiography of the abdomen Computed tomography Measuring blood hormone levels Radioimmunoassay

Computed tomography Explanation: CT or magnetic resonance imaging is used to diagnose the presence and extent of pituitary tumors.

The single modality of pharmacologic therapy for chronic type B viral hepatitis is: Alpha-interferon Hepsera Epivir Baraclude

Alpha-interferon Explanation: Alpha-interferon is a biologic response modifier that is highly effective for treatment of hepatitis B. The other antiviral agents are effective but not the preferred single-agent therapy.

Clinical manifestations of common bile duct obstruction include all of the following except: Amber-colored urine Clay-colored feces Pruritus Jaundice

Amber-colored urine Explanation: The excretion of the bile pigments by the kidneys gives the urine a very dark color. The feces, no longer colored with bile pigments, are grayish, like putty, or clay-colored. The symptoms may be acute or chronic. Epigastric distress, such as fullness, abdominal distention, and vague pain in the right upper quadrant of the abdomen, may occur. If it goes untreated jaundice and pruritus can occur.

A client with Addison's disease comes to the clinic for a follow-up visit. When assessing this client, the nurse should stay alert for signs and symptoms of: calcium and phosphorus abnormalities. chloride and magnesium abnormalities. sodium and chloride abnormalities. sodium and potassium abnormalities.

sodium and potassium abnormalities. Explanation: In Addison's disease, a form of adrenocortical hypofunction, aldosterone secretion is reduced. Aldosterone promotes sodium conservation and potassium excretion. Therefore, aldosterone deficiency increases sodium excretion, predisposing the client to hyponatremia, and inhibits potassium excretion, predisposing the client to hyperkalemia. Because aldosterone doesn't regulate calcium, phosphorus, chloride, or magnesium, an aldosterone deficiency doesn't affect levels of these electrolytes directly.

A nurse expects to note an elevated serum glucose level in a client with hyperosmolar hyperglycemic nonketotic syndrome (HHNS). Which other laboratory finding should the nurse anticipate? Elevated serum acetone level Serum ketone bodies Serum alkalosis Below-normal serum potassium level

Below-normal serum potassium level Explanation: A client with HHNS has an overall body deficit of potassium resulting from diuresis, which occurs secondary to the hyperosmolar, hyperglycemic state caused by the relative insulin deficiency. An elevated serum acetone level and serum ketone bodies are characteristic of diabetic ketoacidosis. Metabolic acidosis, not serum alkalosis, may occur in HHNS.

A client who had developed jaundice 2 months earlier is brought to the ED after attending a party and developing excruciating pain that radiated over the abdomen and into the back. Upon assessment, which additional symptom would the nurse expect this client to have? Hypertension Bile-stained vomiting Warm, dry skin Weight loss

Bile-stained vomiting Explanation: Nausea and vomiting are common in acute pancreatitis. The emesis is usually gastric in origin but may also be bile stained. Fever, jaundice, mental confusion, and agitation may also occur.

Which is a clinical manifestation of cholelithiasis? Epigastric distress before a meal Clay-colored stools Abdominal pain in the upper left quadrant Nonpalpable abdominal mass

Clay-colored stools Explanation: The client with gallstones has clay-colored stools and excruciating upper right quadrant pain that radiates to the back or right shoulder. The excretion of bile pigments by the kidneys makes urine very dark. The feces, no longer colored with bile pigments, are grayish (like putty) or clay colored. The client develops a fever and may have a palpable abdominal mass.

When high levels of plasma calcium occur, the nurse is aware that the following hormone will be secreted: Thyroxine Phosphorus Parathyroid Calcitonin

Calcitonin Explanation: Calcitonin, secreted in response to high plasma levels of calcium, reduces the calcium level by increasing its deposition in the bone.

Which term most precisely refers to the incision of the common bile duct for removal of stones? Choledocholithotomy Cholecystostomy Choledochotomy Choledochoduodenostomy

Choledocholithotomy Explanation: Choledocholithotomy refers to incision of the common bile duct for the removal of stones (liths). Cholecystostomy refers to opening and draining the gallbladder. Choledochotomy refers to opening into the common duct. Choledochoduodenostomy refers to anastomosis of the common duct to the duodenum.

Which condition should a nurse expect to find in a client diagnosed with hyperparathyroidism? Hypocalcemia Hypercalcemia Hyperphosphatemia Hypophosphaturia

Hypercalcemia

When assessing a client with pheochromocytoma, a tumor of the adrenal medulla that secretes excessive catecholamine, the nurse is most likely to detect: a blood pressure of 130/70 mm Hg. a blood glucose level of 130 mg/dl. bradycardia. a blood pressure of 176/88 mm Hg.

a blood pressure of 176/88 mm Hg. Explanation: Pheochromocytoma causes hypertension, tachycardia, hyperglycemia, hypermetabolism, and weight loss. It isn't associated with hypotension, hypoglycemia, or bradycardia.

Which of the following is a characteristic of diabetic ketoacidosis (DKA)? Select all that apply. Elevated blood urea nitrogen (BUN) and creatinine Rapid onset More common in type 1 diabetes Absent ketones Normal arterial pH level

Elevated blood urea nitrogen (BUN) and creatinine Rapid onset More common in type 1 diabetes Explanation: DKA is characterized by an elevated BUN and creatinine, rapid onset, and it is more common in type 1 diabetes. Hyperglycemic hyperosmolar nonketotic syndrome (HHNS) is characterized by the absence of urine and serum ketones and a normal arterial pH level.

Which instruction should be included in the discharge teaching plan for a client after thyroidectomy for Graves' disease? Keep an accurate record of intake and output. Use nasal desmopressin acetate (DDAVP). Have regular follow-up care. Exercise to improve cardiovascular fitness.

Have regular follow-up care. Explanation: The nurse should instruct the client with Graves' disease to have regular follow-up care because most cases of Graves' disease eventually result in hypothyroidism. Annual thyroid-stimulating hormone tests and the client's ability to recognize signs and symptoms of thyroid dysfunction will help detect thyroid abnormalities early. Recording intake and output is important for clients with fluid and electrolyte imbalances but not thyroid disorders. DDAVP is used to treat diabetes insipidus. Although exercise to improve cardiovascular fitness is important, the importance of regular follow-up is most critical for this client.

When the nurse is caring for a patient with type 1 diabetes, what clinical manifestation would be a priority to closely monitor? Hypoglycemia Hyponatremia Ketonuria Polyphagia

Hypoglycemia Explanation: The therapeutic goal for diabetes management is to achieve normal blood glucose levels (euglycemia) without hypoglycemia while maintaining a high quality of life.

A client has been diagnosed with myxedema from long-standing hypothyroidism. What clinical manifestations of this disorder does the nurse recognize are progressing to myxedema coma? Select all that apply. Hypothermia Hypertension Hypotension Hypoventilation Hyperventilation

Hypothermia Hypotension Hypoventilation Explanation: Severe hypothyroidism is called myxedema. Advanced, untreated myxedema can progress to myxedemic coma. Signs of this life-threatening event are hypothermia, hypotension, and hypoventilation. Hypertension and hyperventilation indicate increased metabolic responses, which are the opposite of what the client would be experiencing.

Which is a gerontological consideration associated with the pancreas? Increased amount of fibrous material Increased bicarbonate output Increased rate of pancreatic secretion Increased calcium absorption

Increased amount of fibrous material Explanation: An increase in fibrous material and some fatty deposition occurs in the normal pancreas of people older than 70 years of age. There is a decreased rate of pancreatic secretion and decreased bicarbonate output in older adults. Decreased calcium absorption may also occur.

The nurse assesses a patient who has an obvious goiter. What type of deficiency does the nurse recognize is most likely the cause of this? Thyrotropin Iodine Thyroxine Calcitonin

Iodine Explanation: Oversecretion of thyroid hormones is usually associated with an enlarged thyroid gland known as a goiter. Goiter also commonly occurs with iodine deficiency.

A nurse admits a woman reporting severe right upper quadrant pain after eating dinner. What client risk factors lead the nurse to suspect gallbladder disease? Select all that apply. Multiparous Obese Older than 40 years old Smoking History of diabetes mellitus

Multiparous Obese Older than 40 years old History of diabetes mellitus Explanation: Two to three times more women than men develop cholesterol stones and gallbladder disease; affected women are usually older than 40 years, multiparous, and obese. Diabetes mellitus, cystic fibrosis and frequent weight changes predispose a client to gallbladder disease. Smoking is not a related factor.

The nurse suspects that a patient with diabetes has developed proliferative retinopathy. The nurse confirms this by the presence of which of the following diagnostic signs? Decreased capillary permeability Microaneurysm formation Neovascularization into the vitreous humor The leakage of capillary wall fragments into surrounding areas

Neovascularization into the vitreous humor Explanation: Proliferative retinopathy, an ocular complication of diabetes, occurs because of the abnormal growth of new blood vessels on the retina that bleed into the vitreous and block light. Blood vessels in the vitreous form scar tissue that can pull and detach the retina. Neovascularization into the vitreous humor is considered a diagnostic sign.

A nurse is caring for a client in addisonian crisis. Which medication order should the nurse question? Potassium chloride Normal saline solution Hydrocortisone (Cortef) Fludrocortisone (Florinef)

Potassium chloride Explanation: The nurse should question an order for potassium chloride because addisonian crisis results in hyperkalemia. Administering potassium chloride is contraindicated. Because the client is hyponatremic, an order for normal saline solution is appropriate. Hydrocortisone and fludrocortisone are used to replace deficient adrenal cortex hormones.

A client with status asthmaticus requires endotracheal intubation and mechanical ventilation. Twenty-four hours after intubation, the client is started on the insulin infusion protocol. The nurse must monitor the client's blood glucose levels hourly and watch for which early signs and symptoms associated with hypoglycemia? Sweating, tremors, and tachycardia Dry skin, bradycardia, and somnolence Bradycardia, thirst, and anxiety Polyuria, polydipsia, and polyphagia

Sweating, tremors, and tachycardia Explanation: Sweating, tremors, and tachycardia, thirst, and anxiety are early signs of hypoglycemia. Dry skin, bradycardia, and somnolence are signs and symptoms associated with hypothyroidism. Polyuria, polydipsia, and polyphagia are signs and symptoms of diabetes mellitus.

A patient is admitted to the hospital with possible cholelithiasis. What diagnostic test of choice will the nurse prepare the patient for? X-ray Oral cholecystography Cholecystography Ultrasonography

Ultrasonography Explanation: Ultrasonography has replaced cholecystography (discussed later) as the diagnostic procedure of choice because it is rapid and accurate and can be used in patients with liver dysfunction and jaundice. It does not expose patients to ionizing radiation.

A nurse in the surgical ICU just received a client from recovery following a Whipple procedure. Which nursing diagnoses should the nurse consider when caring for this acutely ill client? Select all that apply. potential for infection acute pain and discomfort alterations in respiratory function fluid volume excess

potential for infection acute pain and discomfort alterations in respiratory function Explanation: Monitor for potential for infection related to invasive procedure and poor physical condition. Monitor for pain related to extensive surgical incision. Monitor for alterations in respiratory function related to extensive surgical incisions, immobility, and prolonged anesthesia. Client is at risk for fluid volume deficit related to hemorrhage and loss of fluids.

A client with cholelithiasis has a gallstone lodged in the common bile duct. When assessing this client, the nurse expects to note: yellow sclerae. light amber urine. circumoral pallor. black, tarry stools.

yellow sclerae. Explanation: Yellow sclerae are an early sign of jaundice, which occurs when the common bile duct is obstructed. Urine normally is light amber. Circumoral pallor and black, tarry stools don't occur in common bile duct obstruction; they are signs of hypoxia and GI bleeding, respectively.

A nurse is completing an assessment of a client with suspected acromegaly. To assist in making the diagnosis, which question should the nurse ask? "Have you had a recent head injury?" "Has your shoe size increased recently?" "Do you experience skin breakouts?" "Is there any family history of acromegaly?"

"Has your shoe size increased recently?" Explanation: Excessive skeletal growth occurs only in the feet, the hands, the superciliary ridge, the molar eminences, the nose, and the chin, giving rise to the clinical condition of acromegaly.

A client with hypothyroidism is afraid of needles and doesn't want to have his blood drawn. What should the nurse say to help alleviate his concerns? "When your thyroid levels are stable, we won't have to draw your blood as often." "It's only a little stick. It'll be over before you know it." "The physician has ordered this test so you can get better sooner." "I'll stay here with you while the technician draws your blood."

"I'll stay here with you while the technician draws your blood." Explanation: The nurse should tell the client that she will stay with him as the blood is drawn. This response provides the client with the reassuring presence of the nurse and enhances the therapeutic alliance, possibly providing a greater opportunity to educate the client. Although telling the client that blood won't need to be drawn as often when thyroid levels are stable provides the client with a rationale for needing blood work, it's more appropriate for the nurse to stay with the client. Saying that the procedure will be over quickly or that the physician has ordered the blood draw ignores the client's stated fear.

A group of students are reviewing information about the relationship of the hypothalamus and the pituitary gland. The students demonstrate the need for additional study when they state which of the following? "The pituitary gland, as the master gland, controls the secretion of hormones by the hypothalamus." "The hypothalamus, a portion of the brain between the cerebrum and brain stem, creates a pathway for neurohormones." "Corticotropin-releasing hormone from the hypothalamus triggers ACTH secretion by the pituitary gland." "The hypothalamus secretes releasing hormones that stimulate or inhibit pituitary gland secretions."

"The pituitary gland, as the master gland, controls the secretion of hormones by the hypothalamus." Explanation: Although the pituitary gland is considered the master gland because it regulates the function of other glands, the hypothalamus influences the pituitary gland. The hypothalamus creates a pathway for neurohormones also known as releasing hormones or factors that stimulate and inhibit secretions from the anterior and posterior lobes of the pituitary gland. Under the influence of the hypothalamus, the lobes of the pituitary gland secrete various hormones. For example, corticotropin-releasing hormone from the hypothalamus causes the anterior pituitary gland to secreted ACTH.

A client has a nasogastric (NG) tube for suction and is NPO after a pancreaticoduodenectomy. Which explanation made by the nurse is the major purpose of this treatment? "The tube helps control fluid and electrolyte imbalance." "The tube provides relief from nausea and vomiting." "The tube allows the gastrointestinal tract to rest." "The tube allows toxins to be removed."

"The tube allows the gastrointestinal tract to rest." Explanation: Postoperative management of clients who have undergone a pancreatectomy or a pancreaticoduodenectomy is similar to the management of clients after extensive gastrointestinal or biliary surgery. An NG tube with suction and parenteral nutrition allow the gastrointestinal tract to rest while promoting adequate nutrition.

A student accepted into a nursing program must begin receiving the hepatitis B series of injections. The student asks when the next two injections should be administered. What is the best response by the instructor? "You must have the second one in 2 weeks and the third in 1 month." "You must have the second one in 1 month and the third in 6 months." "You must have the second one in 6 months and the third in 1 year." "You must have the second one in 1 year and the third the following year."

"You must have the second one in 1 month and the third in 6 months." Explanation: Both forms of the hepatitis B vaccine are administered intramuscularly in three doses; the second and third doses are given 1 and 6 months, respectively, after the first dose.

Health teaching for a patient with diabetes who is prescribed Humulin N, an intermediate NPH insulin, would include which of the following advice? "Your insulin will begin to act in 15 minutes." "You should expect your insulin to reach its peak effectiveness by 12 noon if you take it at 8:00 AM." "You should take your insulin after you eat breakfast and dinner." "Your insulin will last 8 hours, and you will need to take it three times a day."

"You should take your insulin after you eat breakfast and dinner." Explanation: NPH (Humulin N) insulin is an intermediate-acting insulin that has an onset of 2 to 4 hours, a peak effectiveness of 6 to 8 hours, and a duration of 12 to 16 hours. See Table 30-3 in the text.

The nurse is preparing to administer IV fluids for a patient with ketoacidosis who has a history of hypertension and congestive heart failure. What order for fluids would the nurse anticipate infusing for this patient? D5W 0.9% normal saline 0.45 normal saline D5 normal saline

0.45 normal saline Explanation: Half-strength NS (0.45%) solution (also known as hypotonic saline solution) may be used for rehydration of patients with hypertension or hypernatremia and those at risk for heart failure.

A nurse is aware that insulin secretion increases 3 to 5 minutes after a meal and then returns to baseline. If a patient ate breakfast at 7:30 AM, the nurse would expect a baseline level by: 8:30 AM 10:30 AM 12:30 PM 2:30 PM

10:30 AM Explanation: Serum insulin levels return to baseline within 2 to 3 hours.

A nurse is teaching a client with diabetes mellitus about self-management of his condition. The nurse should instruct the client to administer 1 unit of insulin for every: 10 g of carbohydrates. 15 g of carbohydrates. 20 g of carbohydrates. 25 g of carbohydrates.

15 g of carbohydrates. Explanation: The nurse should instruct the client to administer 1 unit of insulin for every 15 g of carbohydrates.

A nurse knows to assess a patient with type 1 diabetes for postprandial hyperglycemia. The nurse knows that glycosuria is present when the serum glucose level exceeds: 120 mg/dL 140 mg/dL 160 mg/dL 180 mg/dL

180 mg/dL Explanation: Glycosuria occurs when the renal threshold for sugar exceeds 180 mg/dL. Glycosuria leads to an excessive loss of water and electrolytes (osmotic diuresis).

Pharmacologic therapy frequently is used to dissolve small gallstones. It takes about how many months of medication with UDCA or CDCA for stones to dissolve? 1 to 2 3 to 5 6 to 12 13 to 18

6 to 12 Explanation: Ursodeoxycholic acid (UDCA [URSO, Actigall]) and chenodeoxycholic acid (chenodiol or CDCA [Chenix]) have been used to dissolve small, radiolucent gallstones composed primarily of cholesterol. Six to 12 months of therapy are required in many clients to dissolve stones, and monitoring of the client for recurrence of symptoms or occurrence of side effects (eg, GI symptoms, pruritus, headache) is required during this time.

Every morning, a client with type 1 diabetes receives 15 units of Humulin 70/30. What does this type of insulin contain? 70 units of neutral protamine Hagedorn (NPH) insulin and 30 units of regular insulin 70 units of regular insulin and 30 units of NPH insulin 70% NPH insulin and 30% regular insulin 70% regular insulin and 30% NPH insulin

70% NPH insulin and 30% regular insulin Explanation: Humulin 70/30 insulin is a combination of 70% NPH insulin and 30% regular insulin.

This example of cholesterol gallstones (left side of picture) is the result of decreased bile acid synthesis and increased cholesterol synthesis in the liver, which in turn, form stones. Cholesterol stones account for what percentage of cases of gallbladder disease in the United States? 15% 25% 60% 75%

75% Explanation: Cholesterol stones account for approximately 75% of cases of gallbladder disease in the United States.

A client is receiving insulin lispro at 7:30 AM. The nurse ensures that the client has breakfast by which time? 7:45 AM 8:00 AM 8:15 AM 8:30 AM

7:45 AM Explanation: Insulin lispro has an onset of 5 to 15 minutes. Therefore, the nurse would need to ensure that the client has his breakfast by 7:45 AM at the latest. Otherwise, the client may experience hypoglycemia.

A client with type 1 diabetes is scheduled to receive 30 units of 70/30 insulin. There is no 70/30 insulin available. As a substitution, the nurse may give the client: 9 units regular insulin and 21 units neutral protamine Hagedorn (NPH). 21 units regular insulin and 9 units NPH. 10 units regular insulin and 20 units NPH. 20 units regular insulin and 10 units NPH.

9 units regular insulin and 21 units neutral protamine Hagedorn (NPH). Explanation: A 70/30 insulin preparation is 70% NPH and 30% regular insulin. Therefore, a correct substitution requires mixing 21 units of NPH and 9 units of regular insulin. The other choices are incorrect dosages for the ordered insulin.

A client with acromegaly is complaining of severe headaches. What does the nurse suspect is the cause of the headaches that is related to the acromegaly? A pituitary tumor A decrease in release in the growth hormone A decrease in the glucose level An increase in cerebral edema

A pituitary tumor Explanation: When the overgrowth is from a tumor, headaches caused by pressure on the sella turcica, a bony depression in which the pituitary gland rests, are common. There is actually an increase in the secretion of the growth hormone. The headaches would not be caused by decreases in glucose levels. The client does not have cerebral edema.

The nurse is reviewing a client's laboratory studies and determines that the client has an elevated calcium level. What does the nurse know will occur as a result of the rise in the serum calcium level? A rise in serum calcium stimulates the release of T lymphocytes. A rise in serum calcium stimulates the release of erythropoietin. A rise in serum calcium inhibits the release of calcitonin. A rise in serum calcium stimulates the release of calcitonin from the thyroid gland.

A rise in serum calcium stimulates the release of calcitonin from the thyroid gland. Explanation: Calcitonin, another thyroid hormone, inhibits the release of calcium from bone into the extracellular fluid. A rise in the serum calcium level stimulates the release of calcitonin from the thyroid gland.

A client with type 1 diabetes is to receive a short-acting insulin and an intermediate-acting insulin subcutaneously before breakfast. The nurse would administer the insulin at which site as the preferred site? Arms Thighs Abdomen Upper buttock

Abdomen Explanation: Although the arms, thighs, and lower back can be used, the preferred site insulin administration is the abdomen which allows more rapid absorption. Reference:

When administering insulin to a client with type 1 diabetes, which of the following would be most important for the nurse to keep in mind? Duration of the insulin Accuracy of the dosage Area for insulin injection Technique for injecting

Accuracy of the dosage Explanation: The measurement of insulin is most important and must be accurate because clients may be sensitive to minute dose changes. The duration, area, and technique for injecting should also to be noted.

What pharmacologic therapy does the nurse anticipate administering when the patient is experiencing thyroid storm? (Select all that apply.) Acetaminophen Iodine Propylthiouracil Synthetic levothyroxine Dexamethasone (Decadron)

Acetaminophen Iodine Propylthiouracil Explanation: Treatments for thyroid storm include the following: a hypothermia mattress or blanket, ice packs, a cool environment, hydrocortisone, and acetaminophen (Tylenol); propylthiouracil (PTU) or methimazole to impede formation of thyroid hormone and block conversion of T4 to T3, the more active form of thyroid hormone; and iodine, to decrease output of T4 from the thyroid gland.

After a thyroidectomy, the client develops a carpopedal spasm while the nurse is taking a BP reading on the left arm. Which action by the nurse is appropriate? Administer a sedative as ordered. Administer IV calcium gluconate as ordered. Start administering oxygen at 2 L/min via a cannula. Administer an oral calcium supplement as ordered.

Administer IV calcium gluconate as ordered. Explanation: When hypocalcemia and tetany occur after a thyroidectomy, the immediate treatment is administration of IV calcium gluconate. If this does not immediately decrease neuromuscular irritability and seizure activity, sedative agents such as pentobarbital may be administered.

The nurse is caring for a patient with hyperparathyroidism and observes a calcium level of 16.2 mg/dL. What interventions does the nurse prepare to provide to reduce the calcium level? Select all that apply. Administration of calcitonin Administration of calcium carbonate Intravenous isotonic saline solution in large quantities Monitoring the patient for fluid overload Administration of a bronchodilator

Administration of calcitonin Intravenous isotonic saline solution in large quantities Monitoring the patient for fluid overload Explanation: Acute hypercalcemic crisis can occur in patients with hyperparathyroidism with extreme elevation of serum calcium levels. Serum calcium levels greater than 13 mg/dL (3.25 mmol/L) result in neurologic, cardiovascular, and kidney symptoms that can be life threatening (Fischbach & Dunning, 2009). Rapid rehydration with large volumes of IV isotonic saline fluids to maintain urine output of 100 to 150 mL per hour is combined with administration of calcitonin (Shane & Berenson, 2012). Calcitonin promotes renal excretion of excess calcium and reduces bone resorption. The saline infusion should be stopped and a loop diuretic may be needed if the patient develops edema. Dosage and rates of infusion depend on the patient profile. The patient should be monitored carefully for fluid overload.

A nurse is teaching a client and the client's family about chronic pancreatitis. Which are the majorcauses of chronic pancreatitis? Malnutrition and acute pancreatitis Alcohol consumption and smoking Caffeine consumption and acute pancreatitis Acute hepatitis and alcohol consumption

Alcohol consumption and smoking Explanation: Alcohol consumption in Western societies is a major factor in the development of chronic pancreatitis, as is smoking. Because heavy drinkers usually smoke, it is difficult to separate the effects of the alcohol abuse and smoking. Malnutrition is a major cause of chronic pancreatitis worldwide, but alcohol consumption is more commonly the cause in Western societies. Caffeine consumption is not related to acute pancreatitis. Acute hepatitis does not usually lead to chronic pancreatitis unless complications develop.

What clinical manifestations does the nurse recognize would be associated with a diagnosis of hyperthyroidism? Select all that apply. A pulse rate slower than 90 bpm An elevated systolic blood pressure Muscular fatigability Weight loss. Intolerance to cold

An elevated systolic blood pressure Muscular fatigability Weight loss. Explanation: Manifestations of hyperthyroidism include an increased appetite and dietary intake, weight loss, fatigability and weakness (difficulty in climbing stairs and rising from a chair), amenorrhea, and changes in bowel function. Atrial fibrillation occurs in 15% of in older adult patients with new-onset hyperthyroidism (Porth & Matfin, 2009). Cardiac effects may include sinus tachycardia or dysrhythmias, increased pulse pressure, and palpitations. These patients are often emotionally hyperexcitable, irritable, and apprehensive; they cannot sit quietly; they suffer from palpitations; and their pulse is abnormally rapid at rest as well as on exertion. They tolerate heat poorly and perspire unusually freely. Reference:

Which intervention should be included in the plan of care for a client who has undergone a cholecystectomy? Placing the client on NPO (nothing by mouth) status for 2 days after surgery Clamping the T-tube immediately after surgery Placing the client in the semi-Fowler position immediately after surgery Assessing the color of the sclera every shift

Assessing the color of the sclera every shift Explanation: If bile is not draining properly, an obstruction is probably causing bile to be forced back into the liver and bloodstream. Because jaundice may result, the nurse should assess the color of the sclerae.

A client has been diagnosed with diabetes and discusses treatment strategies with the nurse. What consequences of untreated diabetes should the nurse include with client teaching? Select all that apply. Blindness Limb amputation Cardiovascular disease Liver failure Kidney disease

Blindness Limb amputation Cardiovascular disease Kidney disease Explanation: The nurse should include blindness, limb amputation, cardiovascular disease, and kidney disease in the teaching of the consequences of untreated diabetes. Liver failure is not a known consequence of diabetes.

A nurse is assigned to care for a patient who is suspected of having type 2 diabetes. Select all the clinical manifestations that the nurse knows could be consistent with this diagnosis. Blurred or deteriorating vision Fatigue and irritability Polyuria and polydipsia Sudden weight loss and anorexia Wounds that heal slowly or respond poorly to treatment

Blurred or deteriorating vision Fatigue and irritability Polyuria and polydipsia Wounds that heal slowly or respond poorly to treatment Explanation: All the options are correct except for weight loss and anorexia. Obesity is almost always associated with type 2 diabetes.

A client with gallstones tells the nurse, "The doctor has to do something. Isn't there something he can give me to dissolve them?" What medication does the nurse know may help dissolve the gallstones? Pancreatin Chenodiol Tacrolimus Cyclosporine

Chenodiol Explanation: Chenodiol suppresses hepatic synthesis of cholesterol and cholic acid to dissolve gallstones. It is administered orally to dissolve gallstones and may require long-term therapy for effectiveness. Pancreatin is a pancreatic enzyme and does not have the properties to dissolve gallstones. Tacrolimus is used to prevent transplant rejection as is cyclosporine.

A patient has been diagnosed with syndrome of inappropriate antidiuretic hormone secretion (SIADH). Which of the following manifestations would be expected in this patient? Concentrated urine Dilute urine Hypernatremia Increased serum osmolality

Concentrated urine Explanation: Because SIADH patients do not excrete dilute urine, the urine osmolality will be increased. Also, serum sodium levels will show low levels because of the retention of urine. There is a decreased serum osmolality with an inappropriately increased urine osmolality.

The primary function of the thyroid gland includes which of the following? Control of cellular metabolic activity Facilitation of milk ejection Reabsorption of water Reduction of plasma level of calcium

Control of cellular metabolic activity Explanation: The primary function of the thyroid hormone is to control cellular metabolic activity. Oxytocin facilitates milk ejection during lactation and increases the force of uterine contraction during labor and delivery. Antidiuretic hormone (ADH) release results in reabsorption of water into the bloodstream rather than excretion by the kidneys. Calcitonin reduces the plasma level of calcium by increasing its deposition in bone.

Hypophysectomy is the treatment of choice for which endocrine disorder? Acromegaly Cushing syndrome Hyperthyroidism Pheochromocytoma

Cushing syndrome Explanation: Transsphenoidal hypophysectomy is the treatment of choice for clients diagnosed with Cushing syndrome resulting from excessive production of adrenocorticotropic hormone (ACTH) by a tumor of the pituitary gland. Hypophysectomy has an 80% success rate.

The nurse is reviewing the laboratory and diagnostic test findings of a client diagnosed with syndrome of inappropriate antidiuretic hormone secretion (SIADH). Which of the following would the nurse expect to find? Elevated serum sodium levels Decreased serum osmolarity Decreased urine sodium levels Elevated urine calcium levels

Decreased serum osmolarity Explanation: With SIADH, serum sodium levels and serum osmolarityare decreased. Urine sodium levels and osmolarity are high. Calcium levels are not involved with this disorder.

A client with long-standing type 1 diabetes is admitted to the hospital with unstable angina pectoris. After the client's condition stabilizes, the nurse evaluates the diabetes management regimen. The nurse learns that the client sees the physician every 4 weeks, injects insulin after breakfast and dinner, and measures blood glucose before breakfast and at bedtime. Consequently, the nurse should formulate a nursing diagnosis of: Impaired adjustment. Defensive coping. Deficient knowledge (treatment regimen). Health-seeking behaviors (diabetes control).

Deficient knowledge (treatment regimen). Explanation: The client should inject insulin before, not after, breakfast and dinner — 30 minutes before breakfast for the a.m. dose and 30 minutes before dinner for the p.m. dose. Therefore, the client has a knowledge deficit regarding when to administer insulin. By taking insulin, measuring blood glucose levels, and seeing the physician regularly, the client has demonstrated the ability and willingness to modify his lifestyle as needed to manage the disease. This behavior eliminates the nursing diagnoses of Impaired adjustment and Defensive coping. Because the nurse, not the client, questioned the client's health practices related to diabetes management, the nursing diagnosis of Health-seeking behaviors isn't warranted.

A client is seeing the physician for a suspected tumor of the liver. What laboratory study results would indicate that the client may have a primary malignant liver tumor? Elevated white blood cell count Elevated alpha-fetoprotein Decreased AST levels Decreased alkaline phosphatase levels

Elevated alpha-fetoprotein Explanation: Alpha-fetoprotein, a serum protein normally produced during fetal development, is a marker that, if elevated, can induce a primary malignant liver tumor. Total bilirubin and serum enzyme levels may be elevated. White blood cell count elevation would indicate an inflammatory response.

A client is admitted to the healthcare facility suspected of having acute pancreatitis and undergoes laboratory testing. Which of the following would the nurse expect to find? Increased serum calcium levels Elevated urine amylase levels Decreased liver enzyme levels Decreased white blood cell count

Elevated urine amylase levels Explanation: Elevated serum and urine amylase, lipase, and liver enzyme levels accompany significant pancreatitis. If the common bile duct is obstructed, the bilirubin level is above normal. Blood glucose levels and white blood cell counts can be elevated. Serum electrolyte levels (calcium, potassium, and magnesium) are low.

which is the primary dietary consideration for a client receiving insulin isophane suspension (NPH) at breakfast? Make sure breakfast is not delayed. Provide fewest amount of carbohydrates at lunch meal. Encourage midday snack. Delay dinner meal.

Encourage midday snack. Explanation: Because NPH is an intermediate-acting insulin that peaks in approximately 4 to 12 hours, a midday snack should be included in daily calorie intake to avoid hypoglycemia. NPH insulin has no immediate effects. Carbohydrates are distributed throughout the meal plan of diabetics to avoid highs and lows. Delaying dinner meal is not indicated with NPH insulin use.

When caring for the patient with acute pancreatitis, the nurse must consider pain relief measures. What nursing interventions could the nurse provide? (Select all that apply.) Encouraging bed rest to decrease the metabolic rate Assisting the patient into the prone position Withholding oral feedings to limit the release of secretin Administering parenteral opioid analgesics as ordered Administering prophylactic antibiotics

Encouraging bed rest to decrease the metabolic rate Withholding oral feedings to limit the release of secretin Administering parenteral opioid analgesics as ordered Explanation: The current recommendation for pain management is the use of opioids, with assessment for their effectiveness and altering therapy if pain is not controlled or increased (Marx, 2009). Nonpharmacologic interventions such as proper positioning (not prone), music, distraction, and imagery may be effective in reducing pain when used along with medications. In addition, oral feedings are withheld to decrease the secretion of secretin.

Which of the following would the nurse most likely assess in a client with diabetes who is experiencing autonomic neuropathy? Skeletal deformities Paresthesias Erectile dysfunction Soft tissue ulceration

Erectile dysfunction Explanation: Autonomic neuropathy affects organ functioning. According the American Diabetes Association, up to 50% of men with diabetes develop erectile dysfunction when nerves that promote erection become impaired. Skeletal deformities and soft tissue ulcers may occur with motor neuropathy. Paresthesias are associated with sensory neuropathy.

When the nurse is caring for a patient with acute pancreatitis, what intervention can be provided in order to prevent atelectasis and prevent pooling of respiratory secretions? Frequent changes of positions Placing the patient in the prone position Perform chest physiotherapy Suction the patient every 4 hours

Frequent changes of positions Explanation: Frequent changes of position are necessary to prevent atelectasis and pooling of respiratory secretions.

Which assessment finding is most important in determining nursing care for a client with diabetes mellitus? Respirations of 12 breaths/minute Cloudy urine Blood sugar 170 mg/dL Fruity breath

Fruity breath Explanation: The rising ketones and acetone in the blood can lead to acidosis and be detected as a fruity odor on the breath. Ketoacidosis needs to be treated to prevent further complications such as Kussmaul respirations (fast, labored breathing) and renal shutdown. A blood sugar of 170 mg/dL is not ideal but will not result in glycosuria and/or trigger the classic symptoms of diabetes mellitus. Cloudy urine may indicate a UTI.

A type 2 diabetic is ordered metformin (Glucophage) as part of the management regime. Which is the best nursing explanation for the action of this drug in controlling glucose levels? Delays digestion of carbohydrates Helps tissues use insulin more efficiently Stimulates insulin release Reduces the production of glucose by the liver

Helps tissues use insulin more efficiently Explanation: Glucophage improves the use of insulin in type 2 diabetes. Alpha-glucosidase inhibitors work by delaying digestion of carbohydrates. Meglitinides stimulates insulin release and/or reduce the production of glucose by the liver.

When the nurse inspects the feet of a diabetic, a tack is found sticking in the sole of one foot. The client denies feeling anything unusual in the foot. Which is the best rationale for this finding? In diabetes, the autonomic nerves are affected. Motor neuropathy causes muscles to weaken and atrophy. High blood sugar decreases blood circulation to nerves. Nephropathy is a common complication of diabetes mellitus.

High blood sugar decreases blood circulation to nerves. Explanation: Diabetic neuropathy results from poor glucose control and decreased blood circulation to nerve tissues. The lack of sensitivity increases the potential for soft tissue injury without awareness. Autonomic neuropathy is a complication of diabetes mellitus but not significant with peripheral injuries. Motor neuropathy does occur with poor glucose control but not specific to this injury. Nephropathy is a common complication that directly affects the kidneys.

The patient admitted with acute pancreatitis has passed the acute stage and is now able to tolerate solid foods. What type of diet will increase caloric intake without stimulating pancreatic enzymes beyond the ability of the pancreas to respond? Low-sodium, high-potassium, low-fat diet High-carbohydrate, high-protein, low-fat diet Low-carbohydrate, high-potassium diet High-carbohydrate, low-protein, low-fat diet

High-carbohydrate, low-protein, low-fat diet Explanation: The nurse should provide a high-carbohydrate, low-protein, low-fat diet when tolerated. These foods increase caloric intake without stimulating pancreatic secretions beyond the ability of the pancreas to respond.

An older adult female client has been complaining of sleeping more, increased urination, anorexia, weakness, irritability, depression, and bone pain that interferes with her going outdoors. Based on these assessment findings, the nurse should suspect which disorder? Diabetes mellitus Diabetes insipidus Hypoparathyroidism Hyperparathyroidism

Hyperparathyroidism Explanation: Hyperparathyroidism is most common in older women and is characterized by bone pain and weakness from excess parathyroid hormone. Clients also exhibit hypercalciuria-causing polyuria. Although clients with diabetes mellitus and diabetes insipidus have polyuria, they don't have bone pain and increased sleeping. Hypoparathyroidism is characterized by urinary frequency rather than by polyuria.

Which of the following is a risk factor for the development of diabetes mellitus? Select all that apply. Hypertension Obesity Family history Age greater of 45 years or older History of gestational diabetes

Hypertension Obesity Family history Age greater of 45 years or older History of gestational diabetes Explanation: Risk factors for the development of diabetes mellitus include hypertension, obesity, family history, age of 45 years or older, and a history of gestational diabetes.

A nurse is providing preoperative teaching to a client undergoing a cholecystectomy. Which topic should the nurse include in her teaching plan? Increase respiratory effectiveness. Eliminate the need for nasogastric intubation. Improve nutritional status during recovery. Decrease the amount of postoperative analgesia needed.

Increase respiratory effectiveness. Explanation: The nurse must teach the client about using an incentive spirometer to promote lung expansion. The high abdominal incision used in a cholecystectomy interferes with respirations postoperatively, increasing the risk of atelectasis. The client will need to use incentive spirometry to promote lung expansion, increase alveolar inflation, and strengthen respiratory muscles. Most clients don't have a nasogastric tube in place after a cholecystectomy. It isn't appropriate to teach improved nutritional status before surgery. It isn't important for the client to be aware of how to decrease the amount of postoperative analgesia, because this is the responsibility of the health care team.

A client is scheduled for a cholecystogram for later in the day. What is the nurse's understanding on the diagnostic use of this exam? It visualizes the gallbladder and bile duct. It visualizes the liver and pancreas. It shows the sizes of the abdominal organs and detects any masses. It visualizes the biliary structures and pancreas via endoscopy.

It visualizes the gallbladder and bile duct. Explanation: The cholecystogram is a diagnostic imaging test used to visualize the gallbladder and bile duct. The celiac axis arteriography visualizes the liver and pancreas. Ultrasonography shows the sizes of the abdominal organs and detects any masses. The endoscopic retrograde cholangiopancreatography (ERCP) visualizes the biliary structures and pancreas via endoscopy.

A client with diabetes mellitus is prescribed to switch from animal to synthesized human insulin. Which factor should the nurse monitor when caring for the client? Polyuria Hypertonicity Low blood glucose concentration Allergic reactions

Low blood glucose concentration Explanation: Clients who switch from animal to synthesized human insulin should initially be monitored for low blood glucose concentrations because the human form of insulin is used more effectively. Human insulin causes fewer allergic reactions than insulin obtained from animal sources. Polyuria and hypertonicity are symptoms of diabetes mellitus.

A client with diabetes is receiving an oral antidiabetic agent that acts to help the tissues use available insulin more efficiently. Which of the following agents would the nurse expect to administer? Metformin Glyburide Repaglinide Glipizide

Metformin Explanation: Metformin is a biguanide and along with the thiazolidinediones (rosiglitazone and pioglitazone) are categorized as insulin sensitizers; they help tissues use available insulin more efficiently. Glyburide and glipizide which are sulfonylureas, and repaglinide, a meglitinide, are described as being insulin releasers because they stimulate the pancreas to secrete more insulin.

A number of pharmacologic agents are used to treat hyperthyroidism. Which of the following drugs is one of the most commonly prescribed and acts by blocking synthesis of the thyroid hormones? Propranolol Dexamethasone Potassium Iodide Methimazole

Methimazole Explanation: Propylthiouracil (PTU) and methimazole are commonly used. They both act by blocking the synthesis of hormones. The other choices suppress the release of the thyroid hormones, except for propranolol which is a beta-adrenergic blocking agent.

A client with pheochromocytoma is scheduled for an adrenalectomy. Which of the following would the nurse perform preoperatively? Begin administering prescribed corticosteroid. Initiate intravenous access for fluid therapy. Monitor blood pressure (BP) frequently. Check for the signs of adrenal insufficiency.

Monitor blood pressure (BP) frequently. Explanation: The nurse should monitor BP frequently before surgery when a client has a pheochromocytoma. When bilateral adrenalectomy is scheduled, the nurse may start IV administration of a solution containing a corticosteroid preparation the morning of surgery. Some surgeons prefer to initiate corticosteroid administration during removal of the adrenals. The nurse monitors for signs of adrenal insufficiency after the surgery.

A client with type 1 diabetes is experiencing polyphagia. The nurse knows to assess for which additional clinical manifestation(s) associated with this classic symptom? Weight gain Muscle wasting and tissue loss Dehydration Altered mental state

Muscle wasting and tissue loss Explanation: Polyphagia results from the catabolic state induced by insulin deficiency and the breakdown of proteins and fats. Although clients with type 1 diabetes may experience polyphagia (increased hunger), they may also exhibit muscle wasting, subcutaneous tissue loss, and weight loss due to impaired glucose and protein metabolism and impaired fatty acid storage.

A client with carcinoma of the head of the pancreas is scheduled for surgery. Which of the following should the nurse administer to the client before surgery? Packed red blood cells Vitamin C Potassium Oral bile acids

Packed red blood cells Explanation: Preoperative preparation includes adequate hydration, correction of prothrombin deficiency with vitamin K, and treatment of anemia to minimize postoperative complications. Parenteral nutrition and blood component therapy are frequently required. Potassium would be given only if the client's serum potassium levels were low. Oral bile acids are not prescribed for a client with carcinoma of the head of the pancreas; they are given to dissolve gallstones. Vitamin C has no implications in the surgery.

Which condition is the major cause of morbidity and mortality in clients with acute pancreatitis? Shock Pancreatic necrosis MODS Tetany

Pancreatic necrosis Explanation: Pancreatic necrosis is a major cause of morbidity and mortality in clients with acute pancreatitis. Shock and multiple organ failure may occur with acute pancreatitis. Tetany is not a major cause of morbidity and mortality in clients with acute pancreatitis.

The nurse is caring for a client with acute pancreatitis who is admitted to the intensive care unit to monitor for pulmonary complications. What is the nurse's understanding of the pathophysiology of pulmonary complications related to pancreatitis? Pancreatitis can elevate the diaphragm and alter the breathing pattern. Pancreatitis causes thickening of pulmonary secretions, impairing oxygenation. Pancreatitis can atrophy the diaphragm and alter the breathing pattern. Pancreatitis causes alterations to hemoglobin, impairing oxygenation.

Pancreatitis can elevate the diaphragm and alter the breathing pattern. Explanation: Aggressive pulmonary care is required for clients with acute pancreatitis. Pancreatitis can elevate the diaphragm and alter the breathing pattern of clients. Pancreatitis can cause retention of pulmonary secretions but does not thicken the secretions. Acute pancreatitis does not alter the hemoglobin.

In actively bleeding patients with esophageal varices, the initial drug of therapy is usually: Sandostatin Inderal Pitressin Corgard

Pitressin Explanation: In an actively bleeding patient, medications are administered initially because other therapies take longer to initiate. Vasopressin (Pitressin) may be the initial mode of therapy, because it produces constriction of the splanchnic arterial bed and decreases portal pressure. It may be administered by IV infusion.

The nurse cares for a client with cholecystitis with severe biliary colic symptoms. Which nursing intervention best promotes adequate respirations in a client with these symptoms? Place the client in semi-Fowler's position. Encourage the client to deep breathe and cough. Instruct the client on the proper use of an incentive spirometer. Encourage the client to ambulate frequently.

Place the client in semi-Fowler's position. Explanation: A client with severe biliary colic is in extreme pain and has a very difficult time taking a deep breath due to severe pain on inspiration. Placing the client in upright or semi-Fowler's position best promotes adequate breathing and best supports the client's function. Ambulation, deep breathing and coughing, and incentive spirometry may be too difficult or impossible for the client with severe biliary colic symptoms.

client who is being tested for syndrome of inappropriate antidiuretic hormone secretion asks the nurse to explain the diagnosis. While explaining, the nurse states that excessive antidiuretic hormone is secreted from which gland? Anterior pituitary Posterior pituitary Adrenal Thyroid

Posterior pituitary Explanation: Antidiuretic hormone is secreted by the posterior pituitary gland.

Which liver function study is used to show the size of the liver and hepatic blood flow and obstruction? Magnetic resonance imaging (MRI) Angiography Radioisotope liver scan Electroencephalography (EEG)

Radioisotope liver scan Explanation: A radioisotope liver scan assesses liver size and hepatic blood flow and obstruction. MRI is used to identify normal structures and abnormalities of the liver and biliary tree. Angiography is used to visualize hepatic circulation and detect the presence and nature of hepatic masses. EEG is used to detect abnormalities that occur with hepatic coma.

A client is admitted to the health care facility with abdominal pain, a low-grade fever, abdominal distention, and weight loss. The physician diagnoses acute pancreatitis. What is the primary goal of nursing care for this client? Relieving abdominal pain Preventing fluid volume overload Maintaining adequate nutritional status Teaching about the disease and its treatmen

Relieving abdominal pain Explanation: The predominant clinical feature of acute pancreatitis is abdominal pain, which usually reaches peak intensity several hours after onset of the illness. Therefore, relieving abdominal pain is the nurse's primary goal. Because acute pancreatitis causes nausea and vomiting, the nurse should try to prevent fluid volume deficit, not overload. The nurse can't help the client achieve adequate nutrition or understand the disease and its treatment until the client is comfortable and no longer in pain.

Which nursing diagnosis is most appropriate for a client with Addison's disease? Risk for infection Excessive fluid volume Urinary retention Hypothermia

Risk for infection Explanation: Addison's disease decreases the production of all adrenal hormones, compromising the body's normal stress response and increasing the risk of infection. Other appropriate nursing diagnoses for a client with Addison's disease include Deficient fluid volume and Hyperthermia. Urinary retention isn't appropriate because Addison's disease causes polyuria.

A nurse assesses a patient diagnosed with hepatic encephalopathy. She observes a number of clinical signs, including asterixis and fetor hepaticus; the patient's electroencephalogram (EEG) is abnormal. The nurse documents that the patient is exhibiting signs of which stage of hepatic encephalopathy? Stage 1 Stage 2 Stage 3 Stage 4

Stage 2 Explanation: The signs listed in the question plus disorientation, mood swings, and increased drowsiness are all indicators of stage 2 hepatic encephalopathy. Refer to Table 25-2 in the text.

After taking glipizide (Glucotrol) for 9 months, a client experiences secondary failure. What should the nurse expect the physician to do? Initiate insulin therapy. Switch the client to a different oral antidiabetic agent. Order an additional oral antidiabetic agent. Restrict carbohydrate intake to less than 30% of the total caloric intake.

Switch the client to a different oral antidiabetic agent. Explanation: The nurse should anticipate that the physician will order a different oral antidiabetic agent. Many clients (25% to 60%) who take glipizide respond to a different oral antidiabetic agent. Therefore, it wouldn't be appropriate to initiate insulin therapy at this time. However, if a new oral antidiabetic agent is unsuccessful in keeping glucose levels at an acceptable level, insulin may be used in addition to the antidiabetic agent. Restricting carbohydrate intake isn't necessary.

Which laboratory test results should a nurse expect to find in a client diagnosed with Hashimoto's thyroiditis? Thyroxine (T4), 22 µg/dl; triiodothyronine (T3), 320 ng/dl; thyroid-stimulating hormone (TSH) undetectable T4, 22 µg/dl; T3, 200 ng/dl; TSH 0.1 mIU/ml T4, 2 µg/dl; T3, 200 ng/dl; TSH 5.9 mIU/ml T4, 2 µg/dl; T3, 35 ng/dl; TSH 45 mIU/ml

T4, 2 µg/dl; T3, 35 ng/dl; TSH 45 mIU/ml Explanation: Normal thyroid function tests are as follows: T4, 5 to 12 µg/dl; T3, 65 to 195 ng/dl; TSH 0.3 to 5.4 mIU/ml. With Hashimoto's thyroiditis, T4 and T3 levels are typically subnormal and TSH is elevated. With primary hyperthyroidism, T4 and T3 levels are elevated and TSH is subnormal. With hypothyroidism, T4 is subnormal and T3 and TSH levels are elevated.

When teaching a client diagnosed with hypothyroidism about medical intervention, which is important for the nurse to communicate? TH may decrease blood glucose concentrations. TH may increase the effect of digitalis preparation. Normal dosages of sedative agents are prescribed. Increased resorption occurs with TH.

TH may increase the effect of digitalis preparation. Explanation: Thyroid hormones may increase the pharmacologic effects of digitalis glycosides, anticoagulant agents, and indomethacin, necessitating careful observation and assessment by the nurse for side effects.

A client with diabetes mellitus develops sinusitis and otitis media accompanied by a temperature of 100.8° F (38.2° C). What effect do these findings have on his need for insulin? They have no effect. They decrease the need for insulin. They increase the need for insulin. They cause wide fluctuations in the need for insulin.

They increase the need for insulin. Explanation: Insulin requirements increase in response to growth, pregnancy, increased food intake, stress, surgery, infection, illness, increased insulin antibodies, and some medications. Insulin requirements are decreased by hypothyroidism, decreased food intake, exercise, and some medications.

Which of the following diagnostic studies definitely confirms the presence of ascites? Ultrasound of liver and abdomen Abdominal x-ray Colonoscopy Computed tomography of abdomen

Ultrasound of liver and abdomen Explanation: Ultrasonography of the liver and abdomen will definitively confirm the presence of ascites. An abdominal x-ray, colonoscopy, and computed tomography of the abdomen would not confirm the presence of ascites.

A female client with chronic hepatitis B has been prescribed recombinant interferon alfa-2b in combination with ribavirin. Which of the following instructions should a nurse provide this client? Avoid calcium-rich foods. Maintain an exercise regimen. Use strict birth control methods. Avoid hot water baths or soaks.

Use strict birth control methods. Explanation: A female client who has been prescribed recombinant interferon alpha-2b in combination with ribavirin should be instructed to use strict birth control methods. This is because ribavirin may cause birth defects. It is not essential for the client to avoid calcium-rich foods or hot baths or soaks. The client needs to maintain physical rest during therapy.

Which of the following is the most effective strategy to prevent hepatitis B infection? Vaccine Barrier protection during intercourse Covering open sores Avoid sharing toothbrushes

Vaccine Explanation: The most effective strategy to prevent hepatitis B infection is through vaccination. Recommendations to prevent transmission of hepatitis B include vaccination of sexual contacts of individuals with chronic hepatitis, use of barrier protection during sexual intercourse, avoidance of sharing toothbrushes, razors with others, and covering open sores or skin lesions.

A client with carcinoma of the head of the pancreas is scheduled for surgery. Which of the following should a nurse administer to the client before surgery? Potassium Vitamin K Vitamin B Oral bile acids

Vitamin K Explanation: Clients with carcinoma of the head of the pancreas typically require vitamin K before surgery to correct a prothrombin deficiency. Potassium would be given only if the client's serum potassium levels were low. Oral bile acids are not prescribed for a client with carcinoma of the head of the pancreas; they are given to dissolve gallstones. Vitamin B has no implications in the surgery.

The nurse is administering Cephulac (lactulose) to decrease the ammonia level in a patient who has hepatic encephalopathy. What should the nurse carefully monitor for that may indicate a medication overdose? Watery diarrhea Vomiting Ringing in the ears Asterixis

Watery diarrhea Explanation: The patient receiving lactulose is monitored closely for the development of watery diarrhea stools, because they indicate a medication overdose. Serum ammonia levels are closely monitored as well.

A client with chronic pancreatitis is treated for uncontrolled pain. Which complication does the nurse recognize is most common in the client with chronic pancreatitis? Weight loss Diarrhea Fatigue Hypertension

Weight loss Explanation: Weight loss is most common in the client with chronic pancreatitis due to decreased dietary intake secondary to anorexia or fear that eating will precipitate another attack. The other answer choices are not the most common complications related to chronic pancreatitis.

A client reporting shortness of breath is admitted with a diagnosis of cirrhosis. A nursing assessment reveals an enlarged abdomen with striae, an umbilical hernia, and 4+ pitting edema of the feet and legs. What is the most important data for the nurse to monitor? Temperature Albumin Hemoglobin Bilirubin

`Albumin Explanation: With the movement of albumin from the serum to the peritoneal cavity, the osmotic pressure of the serum decreases. This, combined with increased portal pressure, results in movement of fluid into the peritoneal cavity. The low oncotic pressure caused by hypoalbuminemia is a major pathophysiologic factor in the development of ascites and edema.

A nursing student has been assigned to care for a client with pancreatic cancer. The student is aware that the risk for pancreatic cancer is most directly proportional to Age Dietary intake of fat Cigarette smoking Presence of diabetes mellitus

age Explanation: Incidence of pancreatic cancer increases with age, peaking in the seventh and eighth decades for both men and women. It is very rare before 45 years, and most patients present in or beyond the sixth decade of life. Cigarette smoking, exposure to industrial chemicals or toxins in the environment, and a diet high in fat, meat, or both are associated with pancreatic cancer, although their roles are not completely clear. Diabetes mellitus, chronic pancreatitis, and hereditary pancreatitis are also associated with pancreatic cancer. Reference:

A client with type 1 diabetes reports waking up in the middle of the night feeling nervous and confused, with tremors, sweating, and a feeling of hunger. Morning fasting blood glucose readings have been 110 to 140 mg/dL. The client admits to exercising excessively and skipping meals over the past several weeks. Based on these symptoms, the nurse plans to instruct the client to administer an increased dose of neutral protamine Hagedorn insulin in the evening. check blood glucose at 3:00 a.m. eat a complex carbohydrate snack in the evening before bed. skip the evening neutral protamine Hagedorn insulin dose on days when exercising and skipping meals.

check blood glucose at 3:00 a.m. Explanation: In the Somogyi effect, the client has a normal or elevated blood glucose concentration at bedtime, which decreases to hypoglycemic levels at 2 to 3 a.m., and subsequently increases as a result of the production of counter-regulatory hormones. It is important to check blood glucose in the early morning hours to detect the initial hypoglycemia.

A client with Cushing's syndrome is admitted to the medical-surgical unit. During the admission assessment, the nurse notes that the client is agitated and irritable, has poor memory, reports loss of appetite, and appears disheveled. These findings are consistent with: depression. neuropathy. hypoglycemia. hyperthyroidism.

depression. Explanation: Agitation, irritability, poor memory, loss of appetite, and neglect of one's appearance may signal depression, which is common in clients with Cushing's syndrome. Neuropathy affects clients with diabetes mellitus — not Cushing's syndrome. Although hypoglycemia can cause irritability, it also produces increased appetite, rather than loss of appetite. Hyperthyroidism typically causes such signs as goiter, nervousness, heat intolerance, and weight loss despite increased appetite.

A client has received a diagnosis of portal hypertension. What does portal hypertension treatment aim to reduce? Select all that apply. fluid accumulation venous pressure blood coagulation fluid output

fluid accumulation venous pressure

A client has been experiencing a decrease in serum calcium. After diagnostics, the physician proposes the calcium level fluctuation is due to altered parathyroid function. What is the typical number of parathyroid glands? four three two one

four Explanation: The parathyroid glands are four (some people have more than four) small, bean-shaped bodies, each surrounded by a capsule of connective tissue and embedded within the lateral lobes of the thyroid.

A client presents with a huge lower jaw, bulging forehead, large hands and feet, and frequent headaches. What is the most reliable method of confirming the client's condition? glucose tolerance test + GH measurement skull radiography alone skull radiography + glucose level MRI + GH measurement

glucose tolerance test + GH measurement Explanation: A glucose tolerance test in combination with a growth hormone measurement is the most reliable method of confirming acromegaly.

A client with adrenal insufficiency is gravely ill and presents with nausea, vomiting, diarrhea, abdominal pain, profound weakness, and headache. The client's family reports that the client has been doing strenuous yard work all day and was sweating profusely. Nursing management of this client would include observation for signs of: hyponatremia and hyperkalemia. hyponatremia and hypokalemia. hypernatremia and hyperkalemia. hypernatremia and hypokalemia.

hyponatremia and hyperkalemia. Explanation: The client's history and presenting symptoms suggest the onset of adrenal crisis. Laboratory findings that support adrenal defficiency and crisis include low serum sodium (hyponatremia) and high serum potassium (hyperkalemia) levels.

A nurse is assigned to care for a postoperative client with diabetes mellitus. During the assessment interview, the client reports that he's impotent and says he's concerned about the effect on his marriage. In planning this client's care, the most appropriate intervention would be to: encourage the client to ask questions about personal sexuality. provide time for privacy. provide support for the spouse or significant other. suggest referral to a sex counselor or other appropriate professional.

suggest referral to a sex counselor or other appropriate professional. Explanation: The nurse should refer this client to a sex counselor or other professional. Making appropriate referrals is a valid part of planning the client's care. The nurse doesn't normally provide sex counseling.

A nurse is caring for a client with diabetes insipidus. The nurse should anticipate administering: insulin. furosemide. potassium chloride. vasopressin.

vasopressin. Explanation: Vasopressin is given subcutaneously to manage diabetes insipidus. Insulin is used to manage diabetes mellitus. Furosemide causes diuresis. Potassium chloride is given for hypokalemia.


Conjuntos de estudio relacionados

Pg. 415 Exam 2 BMCC Using 1. purchases, 2. Cash payments, 3. Sales, 4. Cash receipts, 5. General Journal

View Set

Medical Assisting - Chapter 9 Appointment Scheduling

View Set

English 12B Unit 4: Imitation of Life (Victorian Period, 1837-1901)

View Set

Texas Principles of Real Estate 1: Chapter 4 Terms

View Set

Lesson 2/Chapter 25: Water, Fluid, Electrolytes, and Acid-Base Balance

View Set